[ /tv/ /rf/ /vg/ /a/ /b/ /u/ /bo/ /fur/ /to/ /dt/ /cp/ /oe/ /bg/ /ve/ /r/ /mad/ /d/ /mu/ /cr/ /di/ /sw/ /hr/ /wh/ /lor/ /s/ /hau/ /slow/ /gf/ /vn/ /w/ /ma/ /azu/ /wn/ ] [ Main | Settings | Bookmarks | Music Player ]

No.16169 Reply
File: Безымянный.PNG
Png, 752.00 KB, 725×774 - Click the image to expand
edit Find source with google Find source with iqdb
Безымянный.PNG
Новый физика-тред! И первый вопрос: почему хлеб засыхает при контакте с воздухом.
Предыдущий тред >>41.
>> No.16170 Reply
>>16169
> почему хлеб засыхает при контакте с воздухом.
Потому что из него испаряется влага, очевидно же.
>> No.16171 Reply
>>16170

Но почему когда он в целовановом пакете влага не испряется?
>> No.16173 Reply
>>16171
Потому что она остаётся внутри пакета, а не разлетается по квартире. Очевидно же.
>> No.16175 Reply
>>16171
Няша, это такой изощренный реквест пикчи капитана Пикарда?
>> No.16178 Reply
>>16175
Нет же, я просто пытаюсь выяснить суть разных явлений. И понятий не имею кто такой капитан пикард.
>> No.16278 Reply
>>15976
>>16041
Репост из предыдущего треда
>> No.16323 Reply
Узнать сколько всего времени падало тело, если за последние 2с. до столкновения с землей оно пролетело 60м. Ответ под катом в конце поста. Нужно внятно объяснить как получился этот ответ. Ускорение свободного падения(g) принимаем за 10.


4с.
>> No.16329 Reply
>>16323
Уравнение движения не помогает?
>> No.16338 Reply
>>16329
Помогает. Я туплю/торможу. Поэтому и реквестирую конкретную логическую цепочку.
>> No.16346 Reply
>>16338
%% Я надеюсь, что "падает" предполагает нулевую начальную скорость %%
1. Нарисуй рисунок, а на нем ось координат
2. Из указаного в задаче условия и условия, что тело за нужное тебе время упало на землю получишь два уравнения с двумя неизвестными

Попробуй сейчас
>> No.16347 Reply
>>16323
Пусть скорость тела за две секунды до соприкосновения с землёй v0.
За последние две секунды оно прошло v0t+at^2/2.
v02+102^2/2=60
2*v0+20=60
2*v0=40
v0=20
Чтобы развить скорость в 20 м/с, телу нужно было падать в течение 2 секунд. Итого 4 секунды, считая, что изначально скорость равнялась нулю. Смотрю под спойлер... сошлось.
>> No.16348 Reply
>>16347
Всё время забываю, что форматирование съедает "*". Ну да суть понятна.
>> No.16358 Reply
Так, ну с уравнением понятно. Я точно также получил v0=20, мне не понятно почему при g=10 тело с нуля за 2с пролетело 20м. Это ведь равноускоренное движение, а выглядит как равномерное. Как вообще без формул определяется пройденное расстояние при свободном падении.
>> No.16366 Reply
>>16358
s=gT^2/2, v=gT
>> No.16371 Reply
>>16358
Не путай. Тело с нуля за две секунды с ускорением 10 м/с^2 набрало скорость 20 м/с.
>> No.16376 Reply
Ну не сходится жеж. Если тело в 1-ую секунду прошло 10 метров, во вторую - 20, в 3 - 30, в 4 - 40, то за последние 2с. получается 70м.

Блеать, задачка для 9-го класса.
>> No.16389 Reply
>>16376
Няша, если ускорение десять метров в секунду в квадрате, то тело к концу первой секунды имело скорость 10, в начале её - скорость ноль, средняя скорость пять, итого за первую секунду 5 метров. За вторую 15, за третью 25, за четвёртую 35, 25+35=60.
>> No.16402 Reply
File: internetfistbump.png
Png, 5.77 KB, 284×275 - Click the image to expand
edit Find source with google Find source with iqdb
internetfistbump.png
>>16389
жму руку, бро.
>> No.16529 Reply
Каким образом давление влияет на температуру? И конкретно - температуру воды. Давление чего? Почему температура кристаллизации воды может быть повышена путём уменьшения давления?
>> No.16531 Reply
File: Diag_phase_eau_ru.svg
Svg, 0.03 KB, 391×297 - Click the image to expand
Diag_phase_eau_ru.svg
>> No.16532 Reply
>>16529
Няша, признайся, ты маугли.
>> No.16535 Reply
File: Безымянный12.PNG
Png, 19.97 KB, 674×503 - Click the image to expand
edit Find source with google Find source with iqdb
Безымянный12.PNG
>>16531
Именно это я сейчас и рассматриваю, только вот по такому графику. Я хочу это осмыслить на молекулярном уровне.
>> No.16539 Reply
>>16535
В физике как и везде надо начинать сначала. Подумай, что такое на молекулярном уровне "твёрдое тело", "жидкость", "кристаллизация", "температура", "давление".
>> No.16542 Reply
File: 1232397740005.jpg
Jpg, 61.20 KB, 600×600
Your censorship settings forbid this file.
unrated
>>16539
Тёврдое тело - молекулы не двигаются и только колеюблюстя около положения равновесия?
В жидком уже более подвижны, но не так как в газе?
Температура - кинетическая энергия молекула тела.
Когда температура понижается, то молекулы тела двигаются медленее и постепенно начинают только колебаться около полоджения равновесия - это кристаллизация, переход от жидкого в твёрдое.
Но ведь получается, что температура влияет на давление?
>> No.16544 Reply
>>16529
> Каким образом давление влияет на температуру?
Никаким. Давление в системе влияет на температуру кипения жидкости разве что. Если объем системы постоянен, то по уравнению Клапейрона (pV=nRT) давление влияет на температуру в системе: p = const * T.
> Давление чего?
Давление в термодинамической системе.
> Почему температура кристаллизации воды может быть повышена путём уменьшения давления?
Это следует из диаграммы состояния воды. Диаграмма состояния воды, в свою очередь, зависит от других уравнений. Изучай физичекую химию, если действительно интересно.
>> No.16546 Reply
>>16535
> Именно это я сейчас и рассматриваю, только вот по такому графику. Я хочу это осмыслить на молекулярном уровне.
Далее. Термодинамические понятия, такие как "давление" и "температура" применимы только к макросистемам. Если ты хочешь понять это на молекулярном уровне, займись изучением статистической термодинамики. Но чтобы ее изучить, сперва надо знать классическую.
Да и сомнительно, что у тебя останутся подобные вопросы, если ты будешь знать классическую термодинамику.
>> No.16548 Reply
Что за вассерман насмотрелся аниме и теперь называет всех подряд няшами?
>> No.16571 Reply
>>16539
Как бы ещё осмыслить, почему лёд всплывает на молекулярном уровне )
>> No.16583 Reply
>>16571
Плотность льда меньше плотности воды. Всё.
>> No.16591 Reply
>>16583
И почему же так (на молекулярном уровне)?
>> No.16623 Reply
>>16591

а зачем это объяснять на мол. уровне вообще?
>> No.16686 Reply
>>16623
>>16539- кун сказал, что надо.
>> No.16784 Reply
>>16548

О ком ты?
>> No.16785 Reply
>>16686
Я такого не говорил.
>> No.16787 Reply
Что-то не могу въехать в суть поверхностного натяжения даже по Ландсбергу.
>> No.16832 Reply
Батареи делаю внизу, а не в верху, чтобы комната прогревалась в результате конвекции?
>> No.16841 Reply
>>16832
Да, ты все правильно делаешь.
>> No.16953 Reply
File: Screenshot-Gnuplot (window id : 0).png
Png, 28.63 KB, 1274×490 - Click the image to expand
edit Find source with google Find source with iqdb
Screenshot-Gnuplot (window id : 0).png
Анон, занимающийся моделированием, расскажи, что ты считаешь?
>> No.16984 Reply
>>16953
Вычислительная гидродинамика.
>> No.16991 Reply
>>16984
Можешь привести пример задачи? Ссылки дать? Хочу тоже что-нибудь попробовать
>> No.17028 Reply
Когда в формулах написано что-то типа dV, dP. Это следует понимать как очень малые величины?
>> No.17029 Reply
>> No.17032 Reply
>>16169
Реквестирую способ проведения управляемой термоядерной реакции на дому!!
>> No.17039 Reply
File: 1252474320112.png
Png, 258.57 KB, 584×883 - Click the image to expand
edit Find source with google Find source with iqdb
1252474320112.png
>> No.17047 Reply
File: 73c21a484d8690b5b762e3c56e6f4aa7.png
Png, 1.04 KB, 137×54 - Click the image to expand
edit Find source with google Find source with iqdb
73c21a484d8690b5b762e3c56e6f4aa7.png
Ну вот, допустим, коэффициент объёмного расширения. Какой смысл здесь несёт dv/dt. Типа изменения объёма при изменении температуры? И почему эту же формулу можно написать как дельтаV/дельтаT, т.е. как v2-v1
>> No.17056 Reply
> Какой смысл здесь несёт dv/dt
Производная же.
> И почему эту же формулу можно написать как дельтаV/дельтаT
Потому, что эта формула представляет собой предел >дельтаV/дельтаT
при дельтаТ стремящемуся к нулю, или линейная часть
> изменения объёма при изменении температуры
при фиксированном давлении. см. например http://www.ict.nsc.ru/ru/textbooks/akhmerov/ode_unicode/m-12/m-12.html |
>> No.17089 Reply
>>17047
Это - диффура.
>> No.17090 Reply
>>16991
Луркай Патанкар С. "Численные методы решения задач теплообмена и динамики жидкости."
Но только не перенабранный какими-то уебками pdf с опечатками, а сканированный djvu.
>> No.17110 Reply
>>16323
Внезапно слоупок.
g=10, dt=2, L=60, F''(t)=v'(t)=g;
1)F(t) = 5t^2 + c;
2)F(t0 + dt) - F(t0) = 5(t0)^2 + 20(t0) + 20 + c - 5(t0)^2 - c = 20(t0 + 1);
3)F(t0 + dt) - F(t0) = L = 60 => t0 + 1 = 60/20 = 3 => t0 = 2;
4)t_0 + dt = 2 + 2 = 4.
>> No.17180 Reply
File: 1.PNG
Png, 67.15 KB, 734×712 - Click the image to expand
edit Find source with google Find source with iqdb
1.PNG
Я тут силюсь понять, как работает ручной водяной насос (aka «колонка»). Вот, набросал, как мне представляется его работа. Знающий анон, исправь, пожалйста, ошибки.
>> No.17181 Reply
File: 2.PNG
Png, 63.73 KB, 669×716 - Click the image to expand
edit Find source with google Find source with iqdb
2.PNG
C помощью рычага опускаем поршень вниз, воздух уходит через клапан вверх.
>> No.17185 Reply
File: 3.PNG
Png, 75.25 KB, 1091×706 - Click the image to expand
edit Find source with google Find source with iqdb
3.PNG
Через n итераций, количество воздуха под поршнем уменьшится, т.е. он станет разряженным, соответственно, давление под поршнем уменьшится.
>> No.17186 Reply
File: 4.PNG
Png, 65.23 KB, 783×725 - Click the image to expand
edit Find source with google Find source with iqdb
4.PNG
Жидкость стремится из области повышенного давлением в область с пониженным, и, через клапаны, выходит наружу. Выгода!
>> No.17240 Reply
inb4: тупая школота.

Спутник. Круговая орбита. 600км до поверхности земли. Найти скорость и период обращения. Считать ничего не нужно, только намекнуть с чего начать решение и первую формулу. Спасибо.
>> No.17241 Reply
>>17240
Первая космическая скорость.
>> No.17244 Reply
>>17240
Второй закон Ньютона, закон всемирного тяготения.
>> No.17246 Reply
>>17241
>>17244
Я так понял, что центробежная сила должна уравновешивать силу притяжения. У меня все формулы почему-то завязаны на массе. Аноны помогите, про массу спутника ничего не сказано.
>> No.17256 Reply
>>17246
Скорость и период обращения никак не зависят от массы в данном случае. Если тебе так будет проще, считай, что масса твоего спутника равна 1кг.
>> No.17258 Reply
>>17256
а сила притяжения земли G или g?
>> No.17259 Reply
>>17258
m1a = G * m1*mЗ/r^2
масса спутника сокращается же
А g = (G*m3/r^2) ~ 9.8 на поверхности Земли (все переменные известны), нигде больше она не сработает. А вообще, пиши лучше свои неправильные формулы, а мы исправим.
Ах да r - расстояние между центрами масс, а не поверхностями (ну мало ли).
>> No.17261 Reply
>>17259
как связать 3 закон Ньютона и движение по круговой орбите? Блеять
>> No.17262 Reply
>>17261
центростремительное ускорение вычисли.
>> No.17264 Reply
>>17261
Ох, я такой мимокрокодил, что задачу по диагонали прочитал. >>17261 же, формула a=V^2/R, подставляй в верхнюю. Ну а период легко находится.
>> No.17266 Reply
>>17264
я не могу понять, куда конкретно подставлять. Все эти формулы мне знакомы. Я не могу понять общую картину действия всех сил.
>> No.17267 Reply
>>17261
Если Земля тебя притягивает и ты движешься по круговой орбите вокруг неё*, то с такой же силой ты притягиваешь Землю и она движется по круговой орбите вокруг тебя* * на самом деле вокруг вашего центра масс.
>> No.17268 Reply
>>17267
А формулу можно? С ускорением и пр.
>> No.17269 Reply
>> No.17290 Reply
>>16542
> Температура - кинетическая энергия молекула тела.
Температура это показатель средней энергии движения молекул/атомов вещества. Само собой все молекулы имеют разную энергию движения.
>> No.17292 Reply
Физкач, помоги первокуру, разобраться с дифференциалом в физике.

Написал на физический форум: http://e-science.ru/forum/index.php?showtopic=24552

Прочитай, пожалуйста, и помоги чем можешь.

(переписывать долго, ибо формулы, но не поленись тыкнуться на линк)
>> No.17294 Reply
>>17292
Тебе всё правильно там написали.
>> No.17298 Reply
File: 1242510927062.jpg
Jpg, 32.04 KB, 231×251
Your censorship settings forbid this file.
unrated
Когда после БВ вселенная начала остывать, куда делось тепло? Пошло на что-то другое? Я бы погуглил, но не знаю, как запрос составить.
>> No.17301 Reply
>>17298
> куда делось тепло?
Излучение, образование Очень Тяжелых Элементов.
>> No.17304 Reply
>>17298

Понимаешь в чем проблема, тепло само по себе понятие достаточно земное. В смысл про него имеет смысл говорить когда есть не очень плотные, не очень разреженные, и слегка теплые (300К выше абсолютного нуля- комнатная температура - это вообще ни о чем). Смотри водород в межвездном пространстве ионизован и движется с нехилыми скоростями, если посчитать температуру то формальна она ого го, но ты там замерзнешь. Поэтому особого смысла применять термодинамику для таких систем нет, это только на слегка плотных телах понятие тепла интуитивно понятно и работает.
>> No.17316 Reply
А мне таки кто-нибудь поможет?
>>17180, >>17181, >>17185, >>17186-кун
>> No.17323 Reply
>>17298
пикрилейтенд сделал меня смеятся
>> No.17327 Reply
File: kusomiso.png
Png, 30.33 KB, 176×220 - Click the image to expand
edit Find source with google Find source with iqdb
kusomiso.png
Так. У меня серьезный вопрос. Что наиболее энергетически выгодно: идти по лестнице наступая на каждую ступень или через одну? Казалось бы, в первом случае ты совершаешь больше работы так как движений производится больше, но в тоже время при пропуске каждой второй ступени, ты проходишь расстояния быстрее, переставляя ноги в два раза меньше, но все же давая большую нагрузку на них. Так что же мне делать, анон?
>> No.17329 Reply
File: letchik2.jpg
Jpg, 9.23 KB, 200×200 - Click the image to expand
edit Find source with google Find source with iqdb
letchik2.jpg
>>17301
>>17304
Спасибо, как раз нашел в педивикии такое объяснение.
>> No.17330 Reply
>>17327
самое главное - чтобы твой центр тяжести поднимался равномерно. На остальное вроде пофиг.
Обосновать не могу.
>>17316
В общих чертах ты прав. Но только вода не "стремится из области повышенного давления в область пониженного", а скорее "атмосферный воздух давит на воду вне колонки (ту что под землёй) сильнее, чем на воду под поршнем, и таким образом выталкивает воду". Этим, кстати, объясняется тот факт, что нельзя таким образом (поршневым насосом) поднимать воду на более чем 10 метров - просто силы, создаваемой атмосферным давлением не хватати, какой бы сильный вакуум ты под поршнем не сделал бы.
>> No.17331 Reply
>>17327
фишка в том, что не ясно, на сколько больше ты затрачиваешь силы.
в принципе елси ты просто так прикинешь, то при беге по лестнице ты устанешь намного сильнее чем при неспешном подъеме. так что скорее всего по одной ступени выгоднее.
>> No.17339 Reply
>>17301
Очень тяжелого воорода и гелия?
>>17298
Инфляция (увеличение объёма- а значит и уменьшение температуры)
>> No.17351 Reply
>>17339
> Очень тяжелого водорода и гелия?
Нет, очень тяжелой меди и цинка.
>> No.17371 Reply
>>17327
Надо думать >>17331- анонимус прав. Работа по подъёму складывается из работы против силы тяжести (mgh, зависит от высоты на которую нужно взойти h плюс потери энергии при ходьбе, без разницы- вверх или по горизонтали). По горизонтали ходят обычным шагом, значит и по лестнице проще ходить так же.
Но если лестница невысокая, мне интуитивно кажется что энергетически выгодней разогнаться по горизонтали а потом взбежать по ней, теряя скорость вместо того чтобы идти плавно (я всегда так делаю).
>> No.17372 Reply
>>17327
имхо зависит от пропорций тела к высоте ступеньки, прозреваю что на какой то критичыеской высоте ступеньки затраты энергии резко возрастут.
>> No.17373 Reply
>>17372
алсо

При произвольном темпе ходьбы активность мышц минимальна. Этот феномен объясняется совпадением частоты действия вынуждающих мышечных сил к собственной частоте колебаний нижней конечности.

http://ru.wikipedia.org/wiki/Ходьба_человека
>> No.17402 Reply
>>17327
> наступая на каждую ступень
Ты избежишь лишних потерь энергии на учащенное дыхание и сердцебиение.
>> No.17403 Reply
>>17372
высота ступенек регламентирована строительными нормами. в обычных жилых домах высота ступенек вроде бы не больше 15 см. высота ступенек в театрах и тому подобных местах нанмого меньше.
ступеньки больше 15 см только на аварийных выходах, где крутизна лестницы не играет такой уж роли
>> No.17404 Reply
>>17403
> где крутизна лестницы не играет такой уж роли
ДА ТЫ ШТО!?
>> No.17461 Reply
Я правильно понимаю, что при отсутствии силы терния автомобили не смогли бы остановиться?
>> No.17462 Reply
>>17461
Смогли бы. Используя либо реактивную тягу, либо реверс двигателя.
>> No.17463 Reply
>>17461
> Я правильно понимаю, что при отсутствии силы терния автомобили не смогли бы остановиться?
Это не проблема, так как при отсутствии силы трения они бы не смогли разогнаться :)
>> No.17464 Reply
>>17463
Но она могла бы исчезнуть уже после того, как они поехали.
>> No.17465 Reply
>>17464
> Но она могла бы исчезнуть уже после того, как они поехали.
Вспоминается анекдот про то с какой скоростью должна бежать кошка, чтобы не слышать звука банки привязанной к хвосту.

В общем ты меня понял. :) Здесь нужны инженеры а не математики ;)
>> No.17468 Reply
>>17465
С какой? :3
>> No.17469 Reply
>>17468
400 км/ч
>>17463
Опять же, на реактивной тяге можно разгоняться сколько душе угодно.
>> No.17478 Reply
>>17469
Почему именно такая скорость?
>> No.17507 Reply
>>17478

если долбить твою мамашу в анус, то из ее вопли будут разносится по округе со скоростью

340 м/с = скорость звука
>> No.17510 Reply
>>17507
Во-первых, скорость звука в воздухе составляет 331 м/с, а во-вторых, >>17469-кун ошибся в три раза - результат перевода составляет примерно 1192 км/ч.
>> No.17511 Reply
Добрач, есть один эксперимент: http://ru.wikipedia.org/wiki/Эксперимент_Хафеле_—_Китинга
Почему при разных направлениях полета кинематический вклад берется с разными знаками?
>> No.17533 Reply
File: relative.png
Png, 1.05 KB, 188×52 - Click the image to expand
edit Find source with google Find source with iqdb
relative.png
>>17511
Надо думать, это вклад относительно вращающейся системы отсчёта Земли: для Земли скорость = линейной скорости при вращении, для полёта на восток скорость больше и дельта-т меньше из-за знака минус, а на запад- скорость меньше (а разница будет со знаками - и +).
>>17510
Кошка всё равно услышит звук, передающийся по верёвке и хвосту.
>> No.17534 Reply
Кошка может бежать с любой скоростью в вакууме, очевидно же.
>> No.17536 Reply
>>17534
> Кошка может бежать с любой скоростью
вокруг банки.
>> No.17541 Reply
>>17536
Если взять более реалистичную модель кошки, то это будет автоколебательный процесс (банка будет играть роль отрицательной обратной связи). Ещё у системы "кошка-банка" должна быть устойчивая стационарная точка (когда кошка спит, наличие или отсутствие банки не приводит к её движению). Стационарный режим бега возникает или в присутствии внешнего поля, или при высокой энергии.
>> No.17760 Reply
Расскажите мне вкратце про радиоактивность!
>> No.17764 Reply
>>17760

Ядра атомов состоят из двух видов частиц- протонов и нейтронов. Некоторые комбинации этих частиц неустойчивы. И неустойчивые переходят в устойчивые либо с помощью бета распада (нейтрон разваливается на электрон и протон- электрон вылетает из ядра), либо с помощью альфа распада (из ядра вылетает альфа частица -два протона и два нейтрона), либо просто разваливаясь на две части (так происходит в ядерных реакторах).
   Гамма радиоактивность это испускание высокоэнергетичного кванта света.
>> No.17779 Reply
File: ZN.gif
Gif, 18.98 KB, 351×488 - Click the image to expand
edit Find source with google Find source with iqdb
ZN.gif
> Некоторые комбинации этих частиц неустойчивы
Большинство, я бы сказал. На пикрелейтеде Z- протоны, N- нейтроны.
>> No.17781 Reply
>>17764

Ну в чём суть? почему это опасно для здоровья?
>> No.17783 Reply
>>17781
потому что эти альфа- и бета- частички влетают в тебя, и превращают элементы из которых ты состоишь в какие-то другие. Соответственно, твои клеточки выглядят, как настоящее, но не работают
мимо дилетант
>>17764
А расскажите мне, почему есть альфа-распад, есть бета-распад, но никогда не бывает такого, чтобы из атома улетал один протон (или нейтрон)?
>> No.17798 Reply
File: fusion.png
Png, 1.10 KB, 235×24 - Click the image to expand
edit Find source with google Find source with iqdb
fusion.png
> потому что эти альфа- и бета- частички влетают в тебя, и превращают молекулы из которых ты состоишь в какие-то другие
фикс. Особенно опасно так порвать ДНК, чтобы она оказалась непригодной для восстановления или уничтожения, что способствует возникновению рака. Ну или если излучение сильное, клетки просто разрушаются (что гораздо лучше).
> почему есть альфа-распад, есть бета-распад, но никогда не бывает такого, чтобы из атома улетал один протон
Это энергетически невыгодно, чтобы образовывался водород; выгоднее образование гелия (альфа-частицы).
> (или нейтрон)?
Распад урана же, и прочих тяжелых элементов (да и синтез лёгких тоже).
>> No.17804 Reply
>>17783
потому что эти альфа- и бета- частички влетают в тебя, и превращают элементы из которых

В некотором роде facepalm, потому что электроны способные долететь до ядра должны обладать просто офигительной энергией и по моему в радиоактивных распадах такое не наблюдается.

НО бета (и остальные тоже) частицы, могут учавствовать в образовании свободных радикалов, - считающихся одной из причин старения.

Алсо по энергии, то смертельная доза радиации эквивалентна количеству тепла которое ладонь получает от стакана с чаем, но радиация приложена не к внешней поверхности кожи, а куда попало в том числе и ДНК
>> No.17805 Reply
>>17804
> потому что электроны способные долететь до ядра должны обладать просто офигительной энергией и по моему в радиоактивных распадах такое не наблюдается.
ДА ТЫ ШТО!
Быстраблядь вспомнил опыт Резерфорда.
>> No.17816 Reply
>>17805

Никогда не забывал, опыта по рассеянию АЛЬФА- частиц.

Ну да я посмотрел типичная энергия порядка Mev c пиковыми значениями в несколько десятков. Такое вполне достигается допустим в молниях - и нифига.
>> No.17817 Reply
А я читал, что радиоактивны только элементы, начиная с 83 номера. Тогда причём здесь водород?
>> No.17822 Reply
>>17817
> А я читал, что радиоактивны только элементы, начиная с 83 номера.
Чушь тритий - изотоп водорода, состоящий из протона и двух нейтронов - бета-радиоактивен
>> No.17824 Reply
File: Binding_energy_curve_-_common_isotopes.svg
Svg, 0.04 KB, 671×441 - Click the image to expand
Binding_energy_curve_-_common_isotopes.svg
>>17817
Все элементы, которые отходят от показаной на >>17779 - пикче кривой радиоактивны (а те, которые далеко отходят те и вовсе совершенно нестабильны). Но из стабильных (лежащих на кривой) наиболее устойчива средняя часть- см. пикчу в этом посте. Чем выше энергия связи на нуклон- тем стабильнее элемент, если энергия низкая- то элемент может перейти в более стабильное состояние (ближе к центру) путём распада (справа-налево) или синтеза (слева- направо).
>> No.17835 Reply
File: Безымянный.png
Png, 6.16 KB, 600×460 - Click the image to expand
edit Find source with google Find source with iqdb
Безымянный.png
Сильно-сильно нужен ответ на, якобы, элементарный вопрос. Пикрелейтед. В черном контуре протекает переменный ток. Вообще, он индуцируется от красного контура, к которому подсоединен внешний источник. Дело вот в чем. На участок черного контура накладывается броня - толстостенная металлическая трубка. На рисунке она зеленая. Броня не касается жилы, потому как на жиле - изоляция (синий цвет). Иными словами, прямого электрического контакта брони и контура нет. После наложения брони, общее сопротивление черного контура возрастает. Это надо обосновать на основе закона ома для переменного тока.
***
Вполне очевидно, что броня играет роль емкостного или индуктивного сопротивления, других вариантов нет. Но это кажется мне абсурдом.
>> No.17837 Reply
>>17835

Быдло вариант однако.

Я бы на самом деле посчитал бы L из электродинамики но это тоже не очевидно. А с законом Ома

Однако. Емкостное сопротивление здесь непричем однако.Вроде. То что эта хрень запасает какую то энергию это очевидно. LI^2/2. Так как Энергия больше нуля то можем найти сопротивление которое больше чем то которое было
>> No.17838 Reply
File: Безымянный.png
Png, 126.96 KB, 1024×768 - Click the image to expand
edit Find source with google Find source with iqdb
Безымянный.png
>>17837
В реальности, дело вовсе обстоит так, как на картинке. Нижняя фаза отключена, а трубки, соединенные перемычками - эквивалент медной оболочки жилы. Если броня крепится на верхнюю часть контура оболочки, то ток в ней (оболочке) возрастает, так как часть брони попадает в контур и оттого взаимоиндукция контуров увеличивается. Как сердечник в трансформаторе. Ну а дело в том, что, если положить броню на нижнюю часть, ка на предыдущем рисунке, то ток в оболочке упадет. Упадет, по сравнению с вариантом без брони вовсе. Он не увеличится из-за повышения магнитной проницаемости среды в контуре, потому что броня слишком далеко от жилы с током (нижняя фаза отсоединена). А вот почему уменьшится - такой был вопрос. Препод сказал, забыть о потерях в броне, это незначительно. Сказал, сопротивление контура возрастет. Посмотри, говорит, закон ома и все поймешь)
>> No.17841 Reply
>>17838
Надо думать, броню можно представить как контур, индуктивно связанный с нижней частью схемы, т.е. условно говоря параллельное нагрузке сопротивление; исходный ток разделяется на 2 контура вместо того чтобы идти по одному.
>> No.17845 Reply
File: Безымянный.png
Png, 8.60 KB, 875×402 - Click the image to expand
edit Find source with google Find source with iqdb
Безымянный.png
>>17841
Это сомнительно. Если индуктивная связь, надобно представлять броню в роли катушки. А куда ток течет? Ток в броне, мне кажется, практически полностью движется не по длине, а по толщине. Иными словами, в процессе перемагничивания брони по мере изменения фазы тока. И тогда единственно правильной схемой представления является пикрелейтед. То есть, емкостное замещение. Но в любом случае, препод сказал, потери в броне ничтожны и не должны учитываться, а все к тому и идет.
>> No.17846 Reply
>>17838

Блин слушай прикольно. А система симметрична относительно небольшой добавки индуктивности к одному из контуров ?
В смысле есть две почти одинаковые системы с L1 И L2 и как будет зависеть ток который течет в контуре в зависимости от того к какому из контуров подключен ЭДС
>> No.17867 Reply
File: 1.png
Png, 1.92 KB, 234×99 - Click the image to expand
edit Find source with google Find source with iqdb
1.png
>>17846
Не понял немного. Что будет, если я подам напряжение на жилу, а не на обмотку? Полагаю, все различие будет в разности материала, т.е. сопротивлении контура, в котором ток индуцируется.
> относительно небольшой добавки индуктивности
То есть ток течет в броне по длине с изменением направления тока в контуре? Заряды концентрируются на концах или на внешней и внутренней поверхностях?
>> No.17875 Reply
Почему имеет большой смысл имеет знание скорости в конкретный момент времени? Почему это гораздо важнее, чем скорость на каком-то участке. Ведь в конкретный момент времени, по сути, тело покоится
>> No.17881 Reply
Посоветуйте задачник по физике, где в каждой теме есть как просты задачи, так и посложнее. Причём, с решениями и объяснениями. А то Сивухин пишет ещё вроде ничего, но в задачах просто пиздец какой-то.
>> No.17954 Reply
File: flobots_handlebars2.jpg
Jpg, 251.12 KB, 896×504 - Click the image to expand
edit Find source with google Find source with iqdb
flobots_handlebars2.jpg
Доброфизик объясни, пожалуйста, почему велосипедисты не падают когда едут?
Я знаю, что тут, что-то связанное с моментом импульса. В общем, помоги разобраться.
>> No.17959 Reply
>>17954
Потому же, почему и волчок не падает, когда крутится. Грубо говоря, вращающееся тело устойчивее - чтобы повернуть ось вращения требуется достаточно большая сила.
Хотя тут я не уверен, что велосипед не падает только из-за этого, маловат момент инерции у колеса, во многом, видимо, влияет равновесие ездока. Но опять-таки, стоячий велосипед в равновесии долго не стоит.
>> No.17967 Reply
>>17959
> Потому же, почему и волчок не падает, когда крутится. Грубо говоря, вращающееся тело устойчивее - чтобы повернуть ось вращения требуется достаточно большая сила
Хорошо, тогда изменю вопрос, почему не падает волчок когда крутится? Почему он устойчивее?
Даже если раскрученный волчок отклонить от положения равновесия он не упадет, хотя казалось бы должен. Объясните по подробней, как работает это шайтан-устройство.
>> No.17969 Reply
>>17967
Ну это можно объяснить через уравнение моментов и не очень сложные расчеты с угловыми скоростями, ускорениями и моментами, но это "в цифрах".
Как это объяснить "на словах" я не знаю, так что это такая особенная магия.
Ну или так. Изменение скоростей и ускорений не может происходить мгновенно. Вот ты отклонил волчок на h. Но хитрая машинка взяла, и повернулась, так что твое отклонение на h оказалось сбоку. А ты продолжаешь давить туда же, потому что время твоего действия достаточно велико по сравнению с периодом вращения волчка. Так эта зараза повернется еще за пол периода, и вернет тебе твое давление обратно.
Если ты что-нибудь понел, мы с тобой одинаково безумны.
>> No.17975 Reply
File: e3e078c144ddb8d5b...
Jpg, 207.92 KB, 600×670
edit Find source with google Find source with iqdb
e3e078c144ddb8d5b9477161fe9c1f7b.jpg
File: horo55b796.jpg
Jpg, 146.31 KB, 850×1202
edit Find source with google Find source with iqdb
horo55b796.jpg
File: 1283243375479.jpg
Jpg, 297.96 KB, 708×1000
edit Find source with google Find source with iqdb
1283243375479.jpg

>>17969
Хм, а похоже я понял, что ты хотел сказать.
Что же в любом случае спасибо.
Выбери себе Хоро, что тебе больше по душе же.
>> No.17982 Reply
>>17954 В основном - из-за устройства руля и передней оси. Гироскопические эффекты в колесах, действительно, недостаточны. В одном из номеров "Науки и жизни" за 198какой-то год была хорошая статья по этому поводу, вскоре после её прочтения у меня было ощущение, что я разобрался в вопросе. К сожалению, сейчас я забыл подробности, а доступа к той подшивке у меня сейчас нет, да и гугл-фу подводит. Насколько я помню, автор статьи указывал на то, что при незначительном отклонении от положения равновесия руль выворачивает так, что равновесие восстанавливается. Это обусловлено тем, что вилка велосипеда выгнута вперед. Если бы вилка была ровной и располагалась на оси вращения руля, то такой халявы уже не было бы. А если бы она была загнута назад, то любая случайная флуктуация только усиливалась бы. При этом, правда, улучшаются некоторые другие характеристики (вроде как должен уменьшаться радиус поворота, например), так что оба дизайна реализованы в реальных моделях, на которых автором статьи были проведены эксперименты.
>> No.17985 Reply
>>17982
Автор готов соснуть хуйца у одноколесного велосипеда?
>> No.17994 Reply
File: John+Frusciante.jpg
Jpg, 19.62 KB, 276×512 - Click the image to expand
edit Find source with google Find source with iqdb
John+Frusciante.jpg
Мудрый анон, объясни первокуру-инженеру почему выполняется принцип Гамильтона. В запасе знаний - школьный курс классической механики и начальные знания матана.
Решил в качестве дополнительной литературы к учебнику Савельева начать читать Ландау и Лифшица и столкнулся с совершенно новыми понятиями лагранжиана и действия.
>> No.17995 Reply
>>17994
В силу матанов, вывод принципа наименьшего действия - это математика. Я бы на твоем месте отложил Ландау до курса теормеха.
>>17985
Одноколесный велосипед не столь устойчив, очевидно же. Тут уже должны работать как раз гироскопические эффекты (в меньшей степени, на одноколесном вроде как быстро не разъезжают) и лютый мэдскилл ездока.
>> No.17998 Reply
File: template.jpg
Jpg, 34.39 KB, 274×237 - Click the image to expand
edit Find source with google Find source with iqdb
template.jpg
>> No.17999 Reply
>>17994
Можешь считать принцип наименьшего действия просто другой формулировкой законов Ньютона, или точнее другой формулировкой законов движения. Изначально мы считаем, что система движется так чтобы дейстие было минимально; весь вопрос в том, как построить это действие (конкретно функцию Лагранжа). По Ньютону же мы считаем, что движение каждой материальной точки определяется действием силы, и затем находим эти силы; вся разница только в подходе. Ну и конечно всегда есть априорные предположения или законы
>> No.18013 Reply
File: механика0001.jpg
Jpg, 686.97 KB, 1096×1400 - Click the image to expand
edit Find source with google Find source with iqdb
механика0001.jpg
Реквестирую помощь гуру механики.
>> No.18015 Reply
>>18013
иди в пень!
Тут хоть и доброчан, за тебя никто решать не будет.
Брысь!
>> No.18017 Reply
File: 1261981586733.png
Png, 221.09 KB, 1600×1200 - Click the image to expand
edit Find source with google Find source with iqdb
1261981586733.png
>>18015
Зачем решать, мне и подтолкнуть с решением\формулы подсказать сойдет.
>> No.18024 Reply
File: 1248570251609.png
Png, 33.13 KB, 247×440
Your censorship settings forbid this file.
unrated
>>18017 Самое сложное в этих задачах - прочитать хреново отсканированные условия. Напиши, какие именно проблемы у тебя возникли при решении. Потому что возникает подозрение, что ты просто не пытался решать эти задачи (или поленился ознакомиться с соответствующим разделом учебника и разобрать пару примеров из задачника).
>> No.18040 Reply
Снова про радиоактивность.
Ну вот, допустим, есть атом какого-то вещества. При каких условиях с ним будет происходить альфа-распад? И что с ним произойдт? ПРосто вылетит связка 2-х протонов и нейтронов? И куда она денется? И это ведь один атом. А если какой-то большой кусок вещества.
>> No.18043 Reply
>>18040
Суть такова: альфа-частица, то есть джва протона и джва нейтрона - самая устойчивая комбинация нуклонов. Существует модель, по которой все нуклоны в ядре комбинируются в альфа-частицы, то есть ядро - это наваленная кучка альфа-частиц и двух-трех унылых омег, которым не хватило пар. Модель эта, разумеется, грубая и неточная, но в ядре альфа-частицы появляться любят.
Взаимодействуя друг с другом они могут там набрать приличную скорость (силы там ой-какие большие, такие-то заряды да так плотно сжаты!) и улететь от соседей подальше. Но, разумеется, их удерживает потенциальный барьер - ядерные силы. Энергия альфа-частицы меньше барьера, но барьер не очень толст, а туннельный эффект никто не отменял. Поэтому альфа-частица может свалить из сраного ядра. С очень-очень маленькой вероятностью. Но так как в веществе атомов много, а живут атомы долго, шанс словить частицу от радиоактивного камушка вполне приличный.
>> No.18051 Reply
>>18043
Ну есть вот какой-то элемент и т.е. через какое-то время он станет другим элементом в результате альфа распада?
>> No.18052 Reply
>>18051
Да, в нем начнут появляться примеси другого вещества.
>> No.18055 Reply
File: 065.jpg
Jpg, 121.66 KB, 505×600 - Click the image to expand
edit Find source with google Find source with iqdb
065.jpg
>>18043
> ядро - это наваленная кучка альфа-частиц и двух-трех унылых омег, которым не хватило пар
>> No.18057 Reply
File: chart2000g_rev_E.jpg
Jpg, 24.85 KB, 515×359 - Click the image to expand
edit Find source with google Find source with iqdb
chart2000g_rev_E.jpg
>>18052 Как насчет стабильных изотопов?
>> No.18058 Reply
>>18055
Ололо. А в уране и трансуранах омег (нейтронов) много, и этих нахлебников выкидывают при разделении ядра.
>>18056
А что с ними?
>> No.18059 Reply
>>18057
Всё равно не понял, что с ними.
>> No.18062 Reply
>>18057
Для легких ядер - у них примерно соблюден баланс протонов и нейтронов (это удерживает их от бета-распада, который "старается" уравнивать число протонов и нейтронов) и радиус ядра мал. В силу малости радиуса, альфа-частицы крепко связаны друг с другом и не очень-то стремятся за потенциальный бугор. Точнее, из-за этого сам бугор очень велик. (Вот тут я опять не уверен, лучше спрашивать у учебника, чем у меня. Причем, у школьного.)
>> No.18063 Reply
>>18058
Не нахлебников. Лишние нейтроны поддерживают стабильность в тяжелых ядрах, правда, как именно - я не возьмусь объяснить.
Алсо, то, почему ядра стремятся к равновесию нуклонов, я могу объяснить только из принципа Паули и квантовой механики, но она - такая наука, что "объяснения" в ней звучат не понятнее, чем заклинания.
>> No.18127 Reply
Ускорение и скорость - вектор, но как их представить.
>> No.18143 Reply
>>18127
Скорость - касательная к траектории. А вот ускорение - уже никак. Зачем представлять, считай.
>> No.18175 Reply
>>18143
Ускорение, это вектор, ортогональный скорости и направлению.
>> No.18176 Reply
>>18175
Норкоман же! Равноускоренное движение камня вниз еще никто не отменял.
>> No.18200 Reply
>>16348
> форматирование съедает "*"
Use `` luke!
>> No.18207 Reply
Из соседнего треда:
> Котана попадает в тело, лезвие обляпывается кровью, котана выходит из тела, резко останавливается в конце замаха, кровь с лезвия по инерции улетает дальше.
Почему по инерции? На пятна крови не действуют никакие силы (или скомпенсированы) и поэтому они сохраняют свою скорость, когда оружие останавливается?
>> No.18213 Reply
>>18207
Потому, что капля жидкости не является твердым телом. Ту же скорость, что и катана имеют только жидкие частицы, непосредственно контактирующие с ее поверхностью.
>> No.18221 Reply
>>18213
А, то есть частицы жидкости которые с ней контактировали остаются на катане, остальные - улетают?
>> No.18223 Reply
>>18221
Смотри, взаимодействие между частицами катаны велико, ибо катана суть митол. Взаимодействие между частицами крови меньше - каплю жидкости ты легко можешь размазать пальцем или разделить на две. Точно также невелико и взаимодействие капель крови с лезвием - их можно стереть любой тряпкой.
Когда катана резко останавливается, на все частицы действует огромная сила инерции. (Именно что на каждую частицу, они центральны.) Но частицы катаны крепко связаны друг с другом - катанане рассыпается от таких сил, а вот кровь легко отрывается и летит себе дальше.
>> No.18265 Reply
>>18223
> Когда катана резко останавливается, на все частицы действует огромная сила инерции.
Врёшь!
Сил инерции вообще в природе нет. Это просто удобная абстракция.
Дело всё в том, что когда катана под действием внешних сил (твоих рук) резко останавливается, то на капельки крови не действует почти никаких сил, поэтому-то они по инерции продолжают двигаться в том же направлении, и улетают далеко-далеко (пока не затормозятся трением об воздух или не врежутся в пол/стенку).
>> No.18272 Reply
>>18265
Совершенно верно.
Силу к телу нужно приложить или для придания скорости или прикладывать постоянно для придания ускорения или для противодействия своей бдизняшке (трению).

То есть приложив некоторое время силу к телу (при отсутствии любых других), мы сообщим этому телу постоянную скорость. И скорость эта не изменится, пока не начнёт действовать другая сила (трения, тяготения).

Ft=mV (импульс)
F=ma (ускорение)

Капельки улетают потому что а) им сообщён большой импульс б) силы трения в жидкости и трения об воздух слишком малы, чтобы погасить импульс за столь короткое время (остановка).
>> No.18330 Reply
File: Eagle1.gif
Gif, 16.29 KB, 249×189 - Click the image to expand
edit Find source with google Find source with iqdb
Eagle1.gif
Возможно ли существование в нашем с вами космосе нескольких вселенных? Под «вселенной» я подразумеваю совокупность массы, возникшей в результате одного конкретного большого взрыва.
>> No.18331 Reply
>>18265
Да, но в системе отсчета, связанной с катаной это объяснить много проще. Разве не так?
>> No.18332 Reply
>>18330
Это вопрос, который не может быть решен методами физики. Вопросы вроде "что было до большого взрыва" и "что происходит за границами вселенной" не могут быть исследованы физическими методами.
>> No.18334 Reply
>>18332
Я выразился не совсем корректно. имелось ввиду: возможно ли несколько больших взрывов в одном пространстве?
>> No.18335 Reply
>>18334
Опять некорректно: до большого взрыва не существовало пространство. В общем, это именно тот вопрос, который не имеет физического ответа.
>> No.18336 Reply
>>18335
Несколько больших взрывов, создающих одно и то же пространство?
>> No.18341 Reply
Почему когда читаешь теорию, то всё понятно. А задачи решать не могу.
>> No.18342 Reply
>>18341
Значит, не все понятно.
>> No.18446 Reply
Если бы луна светила в половину от солнца, то ночью бы было светлее?
>> No.18447 Reply
>>18446
Да.
>> No.18459 Reply
>>18335
ЕОТ: есть одна теория, что до большого взрыва был набор частиц - античасиц которых бьыло всегда равное количество и они постоянно аннигилировали повышая общую энергию, но потом ВНЕЗАПНО появилась частица без античастицы, на которую подействовала вся энергия которая накопилась (овер 9000) и потос\м большой взрыв. Следуя этой теории нельзя получить несколько больших взрывов, ибо он происходит при получении ПЕРВОЙ же частицы без античастицы
>> No.18467 Reply
>>18459
А что если появилось одновременно ДВЕ частицы, разнесённые на некоторое расстояние?
>> No.18470 Reply
> появилось одновременно ДВЕ частицы, разнесённые на некоторое расстояние?
Будет две разных несообщающихся вселенных, надо думать. Большой взрыв- то, что запускает классическое время и вызывает расширение пространства из сингулярности, как считает ОТО- либо из квантовой пены, как полагают всякие альтернативные теории. В любом случае до него нормального пространства и времени не было, а было нечто о чём можно только строить беспочвенные предположения. Ну а поскольку мы внутри этой вселенной, то для нас есть только один БВ и второму не бывать.
>> No.18471 Reply
>>18470
Алсо вся тема не слишком отличается от пустых спекуляций.
>> No.18512 Reply
Масса тела, скорость движения которого близка к скорости света в вакууме больше массы того же тела, но покоящегося? Но как только оно остановится масса станет прежней?
>> No.18514 Reply
>>18512
Вопрос обозначений. Тело ведет себя так, словно его масса увеличивается, но куда добавлять множитель - к силам, или к массе - никакой разницы. Насколько я знаю, в современной физике масса полагается постоянной, а Лоренц-фактор перебрасывают в другую сторону. Посмотри формулы, там несложно.
>> No.18650 Reply
Сейчас я открыл окно и было видно, как в него залетают потоки холодного воздуха. Вы, наверное, все видели, как будто что-то расплывается. Но почему воздух визуализируется?
>> No.18651 Reply
>>18650
Изменение давлений и температур слегка меняет и оптические свойства воздуха, как и кривое старое стекло, он начинает искажать изображения. Эффект невелик, но воздуха-то много.
>> No.18659 Reply
> было видно, как в него залетают потоки холодного воздуха
Это частички воды, скорее всего. У тебя за окном температура чуть выше нуля?
>> No.18660 Reply
>>18659

Вроде ниже. Воздух был как будто расплавлен. Такое ещё в фильмах показывают иногда, над асфальтовой дорогой.
>> No.18746 Reply
>>18660
Ну тогда это действительно из-за разницы температур http://ru.wikipedia.org/wiki/Градиентная_оптика . Обычно это летом можно увидеть, особенно если посмотреть на тень. Зимой ни разу не видел.
>> No.18817 Reply
Почему за 1 а.е.м приняли именно 1/12 массы углерода (и при этом какой массы? атом что ли взвесили?), а не 1/14 азота, например?
Катион водорода можно называть протоном водорода? Т.к. в нём всего один электрон и при его удалении (т.е. положительный заряд) остаётся одно ядро, т.е. протон.
>> No.18828 Reply
> Катион водорода можно называть протоном
Да, так и называют %%без "водорода"%
> Почему за 1 а.е.м приняли именно 1/12 массы углерода (и при этом какой массы? атом что ли взвесили?), а не 1/14 азота, например?
Исторически так вышло, насколько я знаю. Судя по википедии это 1/12 массы 12C, а раньше считалось что 1/12 массы "усреднённого природного углерода" (или точнее 1/(12*N_A) массы моля природного углерода), в котором несколько изотопов. Разницу между 1/12 массы 12С и 1/14 массы 14N даст различное соотношение числа протонов и нейтронов и различный дефект масс | http://ru.wikipedia.org/wiki/%D0%94%D0%B5%D1%84%D0%B5%D0%BA%D1%82_%D0%BC%D0%B0%D1%81%D1%81%D1%8B | , но разница будет не очень большой.
>> No.18831 Reply
>>18828
thanks
>> No.18837 Reply
>>18746
подозреваю, разность температур(да, то же, что и над пламенем) + различная влажность уличного и комнатного воздуха, что тоже влияет на коэффициент преломления и искажает лучи
>> No.18847 Reply
File: 1kurs.png
Png, 38.25 KB, 983×207 - Click the image to expand
edit Find source with google Find source with iqdb
1kurs.png
1 курс. Да, я тупой. Но реквестирую помощи. Чисто теоретические вопросы же. Вы должны знать.
На всякий случай ушел читать учебник по данной теме.
>> No.18851 Reply
>>18847
Проще всего написать законы сохранения (энергии и импульса для упругого удара, только импульса + условие слипания для неупругого) и оттуда всё получить. В крайнем случае посмотри например http://www.physics.ru/courses/op25part1/content/chapter1/section/paragraph21/theory.html это
>> No.19028 Reply
File: lightening_block7[1].jpg
Jpg, 34.57 KB, 500×375 - Click the image to expand
edit Find source with google Find source with iqdb
lightening_block7[1].jpg
Привет, физикач.
Суть такова: мне 20 лет, я быдлокодер.
ВНЕЗАПНО понел, что быдлокодинг мне надоел. Хочу бросить свою специальность и стать физиком.
Деньги есть, желание тоже.
Суть такова: как стать ученым-физиком?
>> No.19034 Reply
>>19028

Если есть деньги, то проблем никаких, можешь попробовать либо платно перевестись куда-нибудь, либо заново поступить (егэ сдать - не проблема).
А там уже когда будешь учиться, узнаешь чем можно заняться, но шанс сделать великое открытие очень небольшой, я полагаю.
>> No.19230 Reply
File: Untitled-2.jpg
Jpg, 23.28 KB, 454×59 - Click the image to expand
edit Find source with google Find source with iqdb
Untitled-2.jpg
задача банальна, казалось бы, но что-то меня в ней смущает. Как найти скорость?
>> No.19233 Reply
File: prjam.GIF
Gif, 1.83 KB, 108×135 - Click the image to expand
edit Find source with google Find source with iqdb
prjam.GIF
>>19230
Тень головы... ну, например, рассмотри прямоугольную пирамиду пикрелейтед. ОО' - это фонарь, ab - траектория головы мальчика. Тут мы внезапно замечаем, что ab*2=cd, что доказать нетрудно. Ну а если тень головы прошла вдвое больший путь за тоже время, то и скорость её в 2 раза выше. Ответ - 6 м/с.
>> No.19317 Reply
>>19028
есть деньги -- поступай в какой-нибудь универ в европе. там недорогое обучение и проживание. рашку нахуй шли сразу
>> No.19334 Reply
>>19317

Зря так, классическое физическое образование на высоком уровне во многих вузах России.
>> No.19384 Reply
File: 100kvt-laser.jpg
Jpg, 39.21 KB, 500×374 - Click the image to expand
edit Find source with google Find source with iqdb
100kvt-laser.jpg
Ребята, есть вопросы!
Сегодня на военной кафедре, читали лекцию про лазеры. Вот что я с неё вынес:
1) Лазеры бывают, твердотельные, газовые, жидкие (тут не уверен) и полупроводниковые;
2) КПД самый высокий у полупроводниковых (до 40%), затем идут газовые (до 15%), затем твердотельные (5%). Про жидкие ничего не могу сказать;
3) Мощность при этом, выше всего у газовых, а самая низкая у полупроводниковых;
4) Собственно, главное в лазерах это активный элемент (АЭ) - рубиновый стержень или аргон, к примеру.

Итак, мне стало интересно, бывают ли ещё какие-то типы (виды) лазеров? Расскажите или киньте годных ссылок. Хорошо ли, что высокая мощность? Для каких целей нужна она? Иногда читаю научно-популярную литературу, там пишут, что создают лазеры с очень небольшим показателем мощности (мне кажется, около милливатта, в основном в статьях о медицине), но почему нужна именно такая маленькая мощность, не освещается. Также, мне не понятно, что значит КПД? Ну, точнее, я знаю, что это коэффициент полезного действия, но как определяется он у лазеров, не понял...
И наконец, возможно ли собрать в домашних условиях твердотельный лазер? Меня прям слово рубиновый стержень заманило, это наверно, очень красиво выглядит!

Вот, надеюсь на ваши разъяснения.
>> No.19387 Reply
>>19384 Что касается типов, то начни с этого: http://en.wikipedia.org/wiki/Category:Laser_types
(Also, в русской википедии аналогичная категория тоже есть, хоть и победнее).
Что касается мощности, то помимо неё в лабораторных лазерах иногда "гоняются" за другими параметрами пучка. КПД, насколько я понимаю, определяют как отношение мощности, рассеиваемой в виде лазерного луча, к потребляемой мощности.
А по поводу лазера своими руками - это возможно, только руки нужны прямые. Мой школьный преподаватель по физике в юные годы сам собирал. Вроде как рабочие тела с же напыленными зеркалами сейчас в интернет-магазинах найти можно. Лампы - тоже.
>> No.19388 Reply
>>19387
> с же напыленными зеркалами
c уже напыленными зеркалами
speedfix
P.S. Суровая у вас военная кафедра.
>> No.19390 Reply
> лазера своими руками
Разбери dvd-привод, бумагу, спички, кожу, глаза прожигает.
мимо-не-читал
>> No.19397 Reply
>>19387
Спасибо! С английским, к сожалению, у меня не очень, технические тексты читать тяжело, обилие терминов и так далее. Посмотрю в русской, потом сравню чего нет и поищу отдельно.

P.S. Ну так, военной специальности же учат.

>>19390
Вот зря не читал. Такой лазер не интересует.
>> No.19424 Reply
>>19397
наиболее мощное что можно запилить дома - co2 лазер ватт на 40, (или аж до 200).
трубки, линзы китайцы продают, бп на 15-30кв тоже (но можно и самому собрать), охлаждение вода (аквариумная помпа/автопечка)
тоже мимокрокодил
>> No.19456 Reply
Центростремительное ускорение отвечает за изменение направления скорости. Значит, когда автобус поворачивает, т.е. меняет своё направление, и человека тянет к центру - это и есть то самое ускорение?
>> No.19457 Reply
>>19456
> огда автобус поворачивает
> человека тянет к центру
Ты ничего не путаешь?
Человека будет тянуть наружу - центробежной силой, но так как он движется по дуге, то ускорение у него будет центростремительное. Разница гасится (ну, то что человек остается сидеть в автобусе, а не вылетает в оно на внешнюю сторону поворота) силой реакции опоры и наклоном человек оносительно земли/автобуса.
>> No.19460 Reply
>>19457

Ну я имею в виду, когда стоишь в автобусе (держась за поручень), то при повороте чувствуешь, как тебя тянет вбок (т.е. к центру кривизны, наверное).
>> No.19465 Reply
>>19460
> т.е. к центру кривизны, наверное
Вообще-то, от него.
Во не инерциальных системах отсчета действуют силы инерции, частным случаем которых являются центробежные силы.
В инерциальной системе отсчета если тело движется по кривой, то значит, что какие-то силы сообщают ему центростремительное ускорение.
>> No.19584 Reply
>>19028 -кун снова выходит на связь.

Короче, я поговорил с людьми в самом престижном из доступных мне вузов. Есть 90% инфа, что на западе в среднем лучше, но у нас тоже неплохих физиков готовят, поэтому я думаю все-таки поступать в отечественный вуз.
Сейчас сижу, готовлюсь к тестам.
Думаю поступить на стипендию, ибо в прошлый раз я поступал на быдлокодера и набрал нормально баллов.
Такие дела.
>> No.19612 Reply
>>19584
физиков в массе на бюджет и берут. а если не на бюджет, то значит мозгов мало, и на физфак лучше не соваться.
>> No.19743 Reply
>>19384
Няша, ты ничего не напутал. Только вот как-то мимо прошли жидкостные лазеры, а про них ещё чуть ли не в 80е в какой-то науке и жизни писалось, как собрать такой из "чёрной" лампы, стеклянной трубки, воды, вентилляционной трубы и пары хитрых зеркал. Жидкостные лазеры обладают совершенно вырвиглазными параметрами спектра, фактически в качестве жидкости туда можно сунуть измельченные водоросли, и этот лазер будет вызывать флуоресценцию... в водорослях. В этом прелесть этих лазеров - если некое вещество имеет резонанс в спектре длиннее вынуждающего, то можно вызвать флуоресценцию, используя это самое вещество в качестве рабочего тела. Сейчас твердотельные лазеры почти всегда собирают из готовых зарубежных компонентов, которые довольно дорогие, но доступные. И Бога ради, береги глаза, а то неприятно всю жизнь проходить с отпечатанным на сетчатке спеклом.
>> No.19795 Reply
File: 125676143887272.jpg
Jpg, 300.28 KB, 700×578
Your censorship settings forbid this file.
unrated
Доброанон, есть один бака миниреквест. Я совершенно не шарю в тепломеханике, мне нужен твой мудрый совет по решению сией задачи: Из котла, который имеет одну жаровую трубу, необходимо слить воду. Вычислите время утечки воды, если диаметр жаровой трубы 1м, сечение спускового канала 20мм. Коэфициент расхода - 0.7

Ну или скажи куда мне копать
>> No.20436 Reply
Анон, помоги, никак не могу понять в чем сущность метода Стокса в определении вязкости житкостей. И по пути: что есть динамическая вязкость? Физический смысл ее? Уже вторую неделю не могу на пальцах это обьяснить.
>> No.20460 Reply
File: stokes.gif
Gif, 5.75 KB, 127×296 - Click the image to expand
edit Find source with google Find source with iqdb
stokes.gif
> что есть динамическая вязкость? Физический смысл ее?
> За единицу измерения динамической вязкости принимается вязкость потока жидкости, в которой линейная скорость потока под воздействием давления сдвига в 1 н / м2 имеет градиент в 1 м / с на 1 м расстояния, перпендикулярного к плоскости сдвига.
Смысл в том, что слои жидкости рядом с поверхностью тела движутся со скоростью тела, а слои воды вдали от тела покоятся. Вязкость- это сила трения между слоями. Суть метода Стокса- в определении скорости равномерного движения шарика (или пузырька), на которого действуют сила тяжести, сила Архимеда (постоянные) и сила трения сопротивления жидкости, зависящая от скорости и вязкости. Приравниваем сумму сил нулю, подставляем скорость- получаем вязкость.
>> No.20488 Reply
Как называется операция, когда точки наносят на систему координат и затем проводят между ними линию. Типа устанавливают зависимость между двумя величинами, но как это называется-то? В экселе названо "линия тренда", но в гугле по этому запросу выдаётся совсем другое.
>> No.20520 Reply
>>20488
Это называется "построение графика".
>> No.20523 Reply
>>20520

Ты не понял, что я имел вввиду.
>> No.20540 Reply
>>20488
Аппроксимация.
>> No.20542 Reply
>>20488
Если необходимо просто заполнить промежутки между отдельными точками, то это называется интерполяция.
>> No.20902 Reply
Радиоактивный углерод 14-C имеет период полураспада 5600 лет. Углерод, полученный из живой системы, содержит этот изотоп в количестве, дающем 15,3 распада в минуту на 1 гр. углерода. Сколько распадов следует ожидать от 1 гр углерода, выделенного из куска дерева, сломанного 2000 лет назад?

Я понял ничего, а это же из школьной программы.
Need halp!
>> No.20917 Reply
Почему пустой закрытый пластиковый сосуд сжимается на морозе? Из-за разница давлений воздуха внутри него и на улице? Но ведь давление холодного воздуха меньше, чем тёплогО?
>> No.20920 Reply
>>20917
Ох вей, как всё запущенно.
Внутри тёплый газ. Выносишь на улицу- он охлаждается и сжимается, давление падает. Вот от этого перепада и сжимается.
>> No.20921 Reply
>>20920

Если открыть бутылку, то стенки выравниваются. Почему?
>> No.20922 Reply
>>20921 Из-за упругости стенок и того, что давление в ней выравнивается с атмосферным. До того, как бутылку открывают, силы упругости уравновешивают разницу в давлениях. Когда разница в давлениях, силы упругости распрямляют бутылку обратно (если дело не дошло до пластических деформаций).
>> No.20941 Reply
>>20902
Анон, тут самое главное - понять суть. А суть в том, что пока организм жив, соотношение изотопов углерода в нем не меняется. Как только он умирает (дерево сломали) количество 14-С начинает уменьшаться. И за 5600 лет уменьшится вдвое. И через 5600 лет будет не 15,3 распада в минуту на один грамм, а 7,65. Как именно посчитать, насколько уменьшится количество 14-С за 2000 лет, я не знаю. Подозреваю, что 15,3 - 7,65х2000/5600 = 12,6.
>> No.21210 Reply
File: 1243769791689.jpg
Jpg, 94.61 KB, 745×554
Your censorship settings forbid this file.
unrated
Объясните, что такое энтропия.
>> No.21265 Reply
Вентилятор вращается со скоростью 900 об/мин. После выключения вентилятор, вращаясь равнозамедленно, сделал до отановки 75 оборотов. Сколько времени прошло с момента выключения до остановки?
>> No.21280 Reply
>>21210
В термодинамике, я полагаю? Это величина, бесконечно малое изменение которой dS для системы равно изменению количества тепла дельта-Q к температуре Т. Энтропия для целой системы равна сумме энтропий частей. Согласно второму закону энтропия закрытой системы неубывает.
>> No.21297 Reply
Окей, анончик, вброшу-ка я задачку:
Есть круглая платформа и есть человек, вопрос: на какой угол повернется платформа, если человек по краю плотформы пройдет всю длину окружности? масса платформы = 100 кг, масса человека = 60 кг.
>> No.21300 Reply
>>21280

Ну это как бы понятно, в чём суть? В чём заключается её смысл?
>> No.21334 Reply
>>21300
К термодинамике вообще есть два подхода, феноменологоческий и статистический. >>21280 описал феноменологический подход. Сути в этом нет никакой, и быть не должно. Это просто описание явлений. Мы вводим эту энтропию, так как она описывает то, что мы наблюдаем.

Совсем по-другому в статистической термодинамике. Если ты знаешь кванты, то замечательно, если не знаешь, то наверняка слышал хотя бы, что точно измерить координату и импульс частицы невозможно. Если мы мысленно объединим координату обычного пространства и соответствующую ей координату импульса в одно двухмерное (т.н. фазовое) пространство, то частицу можно одновременно в кусочке фазового пространства площадью не менее чем в постоянную планка. Если мы учитываем все три координаты, то фазовое пространство шестимерно (3 координаты + 3 импульса). Если в системе N частиц, то степеней свободы 3N, и в фазовом пространстве 3N обычных координат и 3N импульсных. И по прежнему объём, в который можно загнать систему, не меньше h^(3N) (не помню, h с чертой или без). Объём h^(3N) называется микросостояние. Мы, конечно, не измеряем координаты всех частиц, а только какую-то совокупность макровеличин вроде объёма, давления и т. д. (макросостояние). Макросостояние содержит много микросостояний, обозначим это число M, причём они входят в него с разной вероятностью (w). Так вот, энтропия - сумма w * ln w по всем микросостояниям. Чем микросостояния более равновероятны и чем их больше, то есть чем меньше нам известно о системе, тем больше энтропия.
>> No.21461 Reply
Господа физики, пожалуйста, помогите решить задачу:
Изменение энтропии в тепловой машине, работающей по циклу Карно, за один цикл равно 10^4 (десять в четвертой степени)Дж/К. Определить полезную работу, если температура нагревателя 320 С, холодильника 20 С.

Могу ли я здесь использовать формулу A=(TхS)-(TнS),где S - изменение энтропии?
>> No.21483 Reply
>>21210
энтропия- мера беспорядочности
>>21297
если смотреть сферически то помоему 0.6пи
>> No.21484 Reply
21483-кун
кнопка с запятой заела
>> No.21486 Reply
>>21483
Скорее 1.2 пи
>> No.21489 Reply
>>21486
нет,именно 0.6
если смотреть совсем сферичски,уровняв массу человека и круга,человек пройдя 2п пути,оборачивает круг на 1 пи,а так как у нас не сферически то получаем 1пи умноженое на соотношение круга и человека.
1Пи*60/100
>> No.21768 Reply
File: 4c66958d5b2b.jpg
Jpg, 12.09 KB, 507×410 - Click the image to expand
edit Find source with google Find source with iqdb
4c66958d5b2b.jpg
Анон, у меня простой вопрос. Имеем пикрелейтед. Пренебрегаем всем, кроме силы тяжести. Известо все, что нужно для решения. Задача - найти ускорения грузов. Я на данный момент могу только сделать систему уравнений, выразить из нее ускорение и т.д. Но мне хочется научиться решать такие задачи, что называется, "с ходу". Ну то есть без составления уравнений в уме и пр. Просто узнать массы грузов и сказать скорость. Это ж элементарно, но власти скрывают.
>> No.21770 Reply
>>21768
Что ещё за простой способ? По твоему рисунку можно условно расположить грузы на прямой, при этом один груз тянет с силой m1g влево, другой с силой m2g вправо, в итоге общая сила прикладываемая к обоим грузам (m1-m2)g, чтобы получить ускорение нужно разделить на массу общей системы m1+m2. Куда ещё проще.
>> No.22217 Reply
При подогреве жидкости снизу тёплые частицы, как менее плотные, поднимаются вверх, но почему ВЕРХНИЕ ХОЛОДНЫЕ ОПУСКАЮТСЯ ВНИЗ?
>> No.22220 Reply
>>22217
Наоборот,холодные,как более плотные и тяжёлые, спускаються вниз, выдавливая тёплые, более рыхлые частички
>> No.22223 Reply
>>22220

Я имел ввиду конвекцию, которая возникает при нагревании снизу.
>> No.22421 Reply
Когда тело движется по инерции, почему оно перестаёт двигаться спустя какое-то время? Только из-за силы трения? Но тело отклонябющееся в автобусе, затем же встаёт ан место.
>> No.22444 Reply
File: BenardConvection.gif
Gif, 95.97 KB, 300×275 - Click the image to expand
edit Find source with google Find source with iqdb
BenardConvection.gif
>>22217
Во-первых, частицы не изменяются при нагревании, но жидкость становится более разреженной. Почему холодная жидкость опускается вниз? Должна же она куда-то деваться.
На самом деле, конвекция - процесс довольно сложный. Не важно, нагреваешь ты снизу или охлаждаешь сверху, это приводит к тому, что снизу теплее, чем сверху. Это состояние энергетически невыгодно и имеет малую энтропию. Но само по себе оне не может стать причиной конвекции. Нужна флуктуация в распределении температур, например, в некоторой точке теплая жидкость поднялась чуть выше, чем вокруг неё. Тогда там тёплая жидкость начнёт подниматься, а вокруг холодная начнёт опускаться, чтобы занять место освобождённое холодной. Если поток тепла поддерживается, начинается циклический процесс. Кстати, такие области называются ячейками Бернара и могут образовывать довольно няшные структуры (пикрилейтед).
>>22421
> Только из-за силы трения?
Именно.
> Но тело отклонябющееся в автобусе, затем же встаёт ан место.
Ты имеешь в виду людей? Ну так люди в общем-то управляют своим движением. А используют они для этого силу трения, держась за поручни, например.
>> No.22446 Reply
>>22444
> такие области называются ячейками Бернара
прошу прощения, Бенара.
>> No.22448 Reply
>>22444
1-Молекулы - нет,связи между ними - даЧастица- несколько молекул.
2-А если я поставлю позади шара магнит? Поверхности шара и пола - идеальные, без трения, шар не остановится?
Всё зависит от условий,если шар котиться под горку по инерции - его остановит силя тяжения
3-А может просто используют силу? Сила трения просто позволяет им найти точку опоры.
>> No.22449 Reply
>>22444

Про ячейки Бенара в курсе, выглядят отвратительно, как по мне. Их вроде можно самостоятельно наблюдать, если тонкий слой масла подогреть, нет? на ютубе видел что-то подобное.
>> No.22484 Reply
Почему человек по разному воспринимает температуру. Ведь когда хъолодно эт овсего лишь меньшая кинетическая энергия молекул. Что это меняет для человека вообще?
>> No.22573 Reply
>>22484
Эта енергия молекул в большей или меньшей степини раздражает рецепторы, поэтому мы ощущаем разность температур.
>> No.22898 Reply
Если бы все тела вокруг передвигались только со скоростью света, чтобы изменилось?
>> No.22899 Reply
>>22898
Всё.
>> No.22911 Reply
няя, хотелось бы поподробнее, ведь если бы все двигались со скоростю свеиа, то относительно друг друга бы когда передвигались ощущали бы это точно так же как и сейчас
>> No.23088 Reply
Все жители в северном полушарии видят одни и те же участки неба? Почему?
Ночью звёзды видно, а днём - нет из-за солнечного света, который рассеивается в атмосфере?
>> No.23094 Reply
>>22911
Тела движущиеся со ск.св. не имеют массы же.
>> No.23101 Reply
>>23094

С чего это???? У электрона есть маса
>> No.23104 Reply
>>23101
А он движется со ск. св????
>> No.23120 Reply
>>23104

А что, не может?
>> No.23121 Reply
>>23120
Нет.
>> No.23122 Reply
>>23121

Даже с близкой к нему скоростью?
>> No.23123 Reply
>>23122
Знаешь, чем меньше масса частицы, тем проще её разгонять. Электрон - сложно.
>> No.23127 Reply
если в ящике летают бабочки то ящик легче, чем когда эта бабочки сядет на его дно?
>> No.23128 Reply
>>23127
Да.
>> No.23163 Reply
>>23128
Смотря как рассматривать, если не учитывать силу ветра от крыльев, то да, если учитывать - может быть по всякому.
>> No.24153 Reply
File: 2426660313_073dea4e96[1].jpg
Jpg, 93.03 KB, 500×375 - Click the image to expand
edit Find source with google Find source with iqdb
2426660313_073dea4e96[1].jpg
Почему запотевают очки в бассейне, пейнтбольная маска и т. п.? Каков принцип работы «незапотевающих» очков?
>> No.24163 Reply
>>23127
Ящик станет тяжелее когда бабочка на дно сядет. Я гарантирую это!
>> No.24164 Reply
>>24153
Влага из воздуха конденсируется на поверхности холодные по отношению к воздуху. Так как в бассейне тепло (ну как правило) и воздух влажный, то и запатевают.
А незапотевающие очки. Точно не знаю, но думаю у них просто стеклышки быстро нагреваются/остывают, влага не успевает конденсироваться.
>> No.24182 Reply
>>24164
Это как-то связано с точкой росы?
>> No.24184 Reply
>>24182
Да, это в некотором смысле и есть точка росы. Но только она выпадает на очки :)
Из википедии:
"Точкой росы при данном давлении называется температура, до которой должен охладиться воздух, чтобы содержащийся в нём водяной пар достиг состояния насыщения и начал конденсироваться в росу." Т.е. воздух рядом с холодными очками охлаждается и излишняя для этой температуры влага из него выпадает росой на очки.
>> No.24368 Reply
File: lkj1009a73jasd7asdw2.png
Png, 2.48 KB, 259×43 - Click the image to expand
edit Find source with google Find source with iqdb
lkj1009a73jasd7asdw2.png
Анон. Объясни тупой первокурсоте. Пикрелейтед третье слагаемое закона сложения скоростей, соответствующее вращательному движению координатных осей. Каким это боком к дифференциированию единичных векторов системы отсчета притесалась угловая скорость? Физический смысл очевиден, но мат. вывод непонятен. Гуглил/английскую википедию читал.
>> No.24374 Reply
>>24368
производная единичного вектора по времени = векторное произведение угловой скорости на этот вектор - смотри в книжке, раздел про векторы и бесконечно малые углы
>> No.24677 Reply
File: Untitled-1.jpg
Jpg, 27.85 KB, 454×340 - Click the image to expand
edit Find source with google Find source with iqdb
Untitled-1.jpg
что ни подставляю, получается либо хуита, либо неправильный ответ.
буду благодарен за расписанное решение
>> No.24683 Reply
>>24677
И где сопротивление среды, жопа ты с ручкой? Угадай, где остановится маятник?
>> No.24693 Reply
>>24683
идеальные условия же, он не остановится. да и не суть, нужно только найти высоту, на которую поднимется эта хуита.
>> No.24694 Reply
>>24693
В идеальных условиях он идеально пролетит все 180 градусов, так шо высота равна радиусу.
>> No.24695 Reply
>>24694
> В идеальных условиях он идеально пролетит все 180 градусов, так шо высота равна радиусу.
Мне кажется, что вы невнимательно глядите на картинку.
> 3m, m, 4m
>>24677
Девять шестнадцатых радиуса?
>> No.24696 Reply
>>24695
Картинку нормальную кто рисовать будет? Управдом? Там шо, три шарика штоле?
>> No.24699 Reply
>>24696
два шарика(3m и m), хрень справа(4m) - это результат абсолютно неупругого удара, h - высота, на которую залетят слепившиеся шарики. ее и надо найти.
>>24694
g там тоже дано. а если бы его там не было(а больше там точно нет никаких сил), он бы так там и висел
>> No.24701 Reply
>>24699
Составь законы сохранения импульса и энергии, няша.
>> No.24706 Reply
>>24701
это я понял. я не понял, что я с ними делаю не так
>> No.24707 Reply
>>24706
Так давай сюда свои писульки штоле.
>> No.24714 Reply
>>24707
там хрень. и я постоянно прихожу к выводу, что h=0.75R
>> No.26761 Reply
Зачем нужен псевдовектор? Угловой скорости, например. Какова польза от него?
>> No.26768 Reply
>>26761
В смысле какая польза? В квантмехе момент импульса - псевдовектор, то есть нельзя одновременно измерить все три его координаты.
>> No.26803 Reply
> Зачем нужен псевдовектор?
Чтобы отличать его от истинного (полярного) вектора, который изменяет своё направление при обращении координат. Аналогично есть скаляры и псевдоскаляры. К примеру энергия должна быть истинным скаляром, и если у тебя она зависит от псевдоскаляра, по условиям симметрии (не важно, правая система координат или левая) в простейшем случае должна быть либо вторая степень, либо нужно домножить на другой псевдоскаляр.
>> No.26805 Reply
Алсо пример: при плоском вращении угловая скорость омега- псевдоскаляр, энергия связанная с вращением не может быть пропорциональна омега, а только второй степени (I w^2/2).
>> No.26876 Reply
Ничего не понял, но видимо действительно нужен.
Другой вопрос, если вскипятить минеральную воду до её полного испарения, на дне кружки останутся твёрдые соли?
>> No.26877 Reply
>>26876
Да.
Если на морозе разливать горячую и холодную воду, какая будет иметь более ровную поверхность?
>> No.26894 Reply
>>26877
От многого зависит, я полагаю. И вообще, у тебя есть возможность проверить это на практике.
>> No.26919 Reply
>>26894
Угу, де заливать? Во дворе?
Боюсь от двориников и соседей пиздюлину получить. Пичалька.
Новый вопрос: есть огромная микроволновка, есть человек, есть фольга.
Если человека просто начать готовить в микроволновке - пизда ему.
Если человек сделает костюм из фольги, с ним что-то произойдёт?
кому не жалко микроволнушку, проведите такой опыт на крысках, мне просто жалко микроволновку и крыс
>> No.27031 Reply
Если кусок хлеба и печенье положит вместе в пакет, чтобы они соприкасались, то часть печенья становится влажной. Чем это объяснить? Испарением влаги из хлеба и её же конденсацией в печенье?
>> No.27047 Reply
>>27031
Чё? Наркоман чтоли сука?
Любая система стремиться к равновесию, в данном случае равновесие наступает, когда всё внутри пакета приобретает одну и ту же влажность. Если вы положите сухарик и влажную печеньку - сухарик намокнет а печенька высохнет.
чё за хуйня? откуда у вас такой бред в головах берётся?
>> No.27080 Reply
>>27047
С температурой да, так. Про влажность не слышал такого. По капиллярами что ли она перебирается?
>> No.27081 Reply
И что, вода из хлеба не может испаряться?
>> No.27147 Reply
File: хуй.JPG
Jpg, 21.02 KB, 509×494 - Click the image to expand
edit Find source with google Find source with iqdb
хуй.JPG
>>27081
В задании дана фотография струи воды, вытекающей из шланга (я схематически изобразил на рисунке), расстояние между клеточками 10 см. Нужно найти начальную скорость. Это график скорости от расстояния или скорости от времени? Потому как найти начальную скорость, это движение под углом к горизонту, следовательно есть и вертикальная и горизонтальная скорости.
>> No.27151 Reply
>>27147 Норкома~н? Это график в координатах х,y, где ось х направлена из верхнего левого угла направо, а ось y - вниз. По твоему рисунку похоже на то, что струя изначально вытекает горизонтально. В таком случае движение частиц воды описывается уравнениями x=vt, y=gt²/2, где v - начальная скорость. Откуда несложно эту самую скорость выразить: v=√gx²/2y.
>> No.27167 Reply
>>27151
а координаты х и у откуда брать?
>> No.27168 Reply
>>27167
по графику
>> No.27174 Reply
Анончики, прошу помощи!
Собираюсь поступать на второе высшее, посему посоветуйте хороший учебник по физике для гуммунитарея, который охватывал бы весь школьный курс. Что-нибудь типа "пособия для поступающих в ВТУЗы" или школьного справочника, их так много, что сам не могу выбрать. Заранее спасибо.
>> No.27179 Reply
>>27174
ландсберг "элементарный учебник физики"
вот тебе ещё сайт http://class-fizika.narod.ru/
>> No.27725 Reply
Почему скорость звука в воздухе зависит от температуры? Это связано с больше кинетической энергией молекул газов в тёплом воздухе?
>> No.27743 Reply
File: image002.jpg
Jpg, 13.56 KB, 388×107 - Click the image to expand
edit Find source with google Find source with iqdb
image002.jpg
Физик-кун, есть ли какие-нибудь методические пособия по решению задач для вузов. Пояснения как решать с примерами.
Мне не понятно как правильно решается большая часть задач из пикрелейтеда.
>> No.27767 Reply
File: wallpaper-700434.jpg
Jpg, 538.36 KB, 2000×1517 - Click the image to expand
edit Find source with google Find source with iqdb
wallpaper-700434.jpg
Физик-куны, объясните пожалуйсто неучу, как так называемая Теория Всего (ТОЕ) сочетается с Принципом неопределенности гейзенберга? Они же прямо противоречат друг другу по сути. Или все дело в допустимых в ТОЕ погрешностях измерений при расчете всех взаймодействий одновременно? И кстати, выше ли эта погрешность чем постоянная Планка?
>> No.27809 Reply
почему человек не ощущает суточного вращения земли?
если просверлить землю насквозь, то окажешься в другом полушарии, как там ходят люди?
>> No.27834 Reply
>>27809

Зомбоящик врет - мир плоский. Я свисал с его края, видел китов и черепаху.
>> No.27840 Reply
>>27767
> Теория Всего
Неплохо было бы уточнить, какую из претендующих на это звание теорий ты имеешь в виду.
> гейзенберга
Имена собственные нужно писать с большой буквы.

  Что-то подсказывает мне, что квантовую физику ты еще не проходил, и являешься просто энтузиастом. Если это так, то неплохо бы тебе ознакомиться с третьим томом курса теоретической физики Ландау и Лифшица. Правда, некоторое время потребуется на то, чтобы к обозначениям привыкнуть.
  P.S. Прости, если получилось грубовато. Просто не хочется своими несовершенными познаниями случайно посеять в твоем сознании новые заблуждения.
>> No.27841 Reply
File: 1248294506197.jpg
Jpg, 141.96 KB, 577×875
Your censorship settings forbid this file.
unrated
>>27809 Потому что в не инерциальной системе отсчета, связанной с вращающейся Землей на человека действует сила, равная по модулю mω²r, где ω - угловая скорость вращения Земли, r - расстояние от человека до оси вращения, m - его масса. Отношение модуля этой силы к модулю силы тяжести: mω²r/mg=ω²r/g, что даже на экваторе даст всего-то ~0.003.
Люди держатся на Земле потому, что наибольшее притяжение на них оказывает (сюрприз!) планета массой почти в 6•10²⁴ кг, по поверхности которой они разгуливают. К примеру, Луна тащит к себе с силой, относящейся к силе притяжения Земли как Fл/Fз=(Mл/Mз)•(Rз/Rл), где Мл и Mз - массы Луны и Земли, а Rл и Rз - расстояния от рассматриваемого человека до центров этих объектов. Подставив численные данные, получим для Луны в перигее величину около 4,1•10⁻⁶. Даже вращение Земли ощутить в тысячу раз проще. Такие дела. Силу притяжения какого-нибудь человека к тебе лучше и не пытайся посчитать - это слишком плохо сказывается на самооценке.
>> No.27847 Reply
File: Konachan.com - 85335 sample.jpg
Jpg, 1695.54 KB, 1920×1200 - Click the image to expand
edit Find source with google Find source with iqdb
Konachan.com - 85335 sample.jpg
>>27840
За Гейзенберга виноват, каюсь. Эт надо ж было так неуважительно к великому человеку. По поводу теории всего - я имею ввиду ту, что в научной литературе называют "Единой теорией поля". На сабж можешь забить, ибо покурив эту "теорию" я обнаружил, что она является утопическим обобщением конца 19 века, попыткой обьяснения на основе электромагнетизма всех известных физических явлений. В двадцатом веке не выдержала ни одной проверки экспериментом. А, ну и конечно же, была разбита в пух и прах после того, как Гейзенберг вывел принцип неопределенности, а Планк свою постоянную, которая определяла погрешность, меньше которой измерения не могут быть в принципе.


P.S. Желаемое издание рекомендованной тобой книги укажи пожалуйсто. И за какой год оно наиболее полное. Я почитаю.:3
>> No.27859 Reply
File: Belonuchkin_OsnoviFizikiKursObshejFiziki.rar
Rar, 5.49 KB, 0 files - Click the image to get file
view
Belonuchkin_OsnoviFizikiKursObshejFiziki.rar
>>27847 Сам Планк постоянную, позднее названную в его честь, называл "квантовой постоянной", и вводил её для того, чтобы связать энергию кванта с его частотой. Но ты навел меня на неплохую мысль. Существенно помогает прояснить понимание тех или иных эффектов изучение истории их открытия. Хорошо, когда параллельно с сухим остатком излагают также историю развития научной мысли в начале двадцатого века. Так что посоветую-ка я лучше что-нибудь вроде прикрепленного учебника или вот этой книги: http://fizmatkniga.ru/catalog/st-eae2e0edf2eee2e0ff20ece8eaf0eef4e8e7e8eae0/product-845/
  А что касается курса Ландау и Лифшица, то какого-то определенного издания не посоветую. Сам обычно стараюсь с последними изданиями книг работать - просто потому, что в них меньше опечаток и они лучше сохранились.
>> No.27862 Reply
Краткий справочник для инженеров и студентов Высшая математика, физика, теоретическая механика, сопротивление материалов. А.Д. Полянин.rar
>>27859
Благодарю вас, товарищ. И за пояснение, и за предоставленную литературу. Будь в интернетах поболее склонных к диалогу людей, наш анонимный социум избежал бы многих неприятных казусов. Отблагодарил бы вас талонами на джоули или кванты света, но такой валюты не запилили ,увы, так что пусть будет справочник формул. Он немного нубский, но больше мне, увы, предложить нечего.
>> No.28184 Reply
Когда смотришь вечером из окна на улицу, свет фонарей как бы растягивается около источников света. В чём суть явления?
>> No.28191 Reply
File: 1292692036355.jpg
Jpg, 61.48 KB, 320×240
edit Find source with google Find source with iqdb
1292692036355.jpg
File: 65.JPG
Jpg, 22.84 KB, 386×789
edit Find source with google Find source with iqdb
65.JPG

Приветствую, физикач, школьник-уёбок ITT.
Суть такова: решил я, значит, внезапно сдавать ЕГЭ по физике. Решил, ну и начал её усердно учить: курить учебники, решать задачи, etc. И вот, за месяц усердной зубрёжки, я выявил две темы, решать задачи на которые я совершенно не умею, даже после детального разбора примеров. Посему прошу твоей помощи, доброанон, с двумя такими задачками: реши, объясни ход своих мыслей по возможности, соображалка то у меня работает, просто нет развитого скилла, такие дела. С меня, естественно, причитается.


Две параллельные неподвижные диэлектрические пластины расположены вертикально и заряжены разноимённо. Пластины находятся на расстоянии d=2см друг от друга. Напряжённость поля в пространстве внутри пластин равна Е=4*100000 В/м. Между пластинами на равном расстоянии от них расположен шарик с зарядом q=(10 в степени -10)Кл и массой m=20мг. После того как шарик отпустили он начинает падать и ударяется об одну из пластин. На сколько уменьшится высота местонахождения шарика (дельта h)к моменту его удара оь одну из пластин?

Предположим, что схема энергетических уровней атомов некоего вещества имеет вид, показанный на рисунке, и атомы находятся в состоянии с энергией W1. Электрон, столкнувшись с одним из таких атомов, отскочил, приобретя некоторую дополнительную энергию. Импульс электрона после столкновения с покоящимся атомом оказался равным 1.2(10 в степени -24) кгм/с. Определите кинетическую энергию электрона до столкновения. Возможность испускания света атомами при столкновении с электроном пренебречь.
>> No.28192 Reply
>>28184
Смотришь через занавеску - дифракция света.
>> No.28199 Reply
>>28192
почему? свет как-то огибает дырочки в занавеске?
>> No.28202 Reply
Уважаемые, будьте добры, помогите с задачей, суть такова:
К двум диэлектрическим пластинам проницаемостью 7 и 5 приложена разность потенциалов 1200В. Найти плотность связанных зарядов на границе раздела сред, если известно, что толщина обоих пластин равна 1см.
Ответ: 17.7 нКл/м^2
Сколько и как ни решаю так и не получается.
>> No.28203 Reply
>>28191
Первая задача.надеюсь на вашу "соображалку", о коей вы непременули сообщить, посему не расписываю во всех подробностях
На шарик действуют две силы: со стороны поля F=Eq и сила притяжения земли F=mg. Выражаем горизонтальную составляющую ускорения и получаем a=(Eq)/m, зная расстояние до пластины, находим время движения по формуле s=(at^2)/2. В последнюю формулу подставляем g и t получаем S собственно ответ.
первокурсота-уёбок из предыдущего поста:3
>> No.28214 Reply
>>28203
А сами пластины не притягиваются, разве?
>> No.28217 Reply
>>28214
Прочти условие еще раз. Прочитал? Еще разок, внимательно. Все ясно, няша?
>> No.28368 Reply
File: Simple_harmonic_motion_animation[1].gif
Gif, 24.85 KB, 512×242 - Click the image to expand
edit Find source with google Find source with iqdb
Simple_harmonic_motion_animation[1].gif
>> No.28370 Reply
>>28368
И к чему это?
>> No.28371 Reply
>>28368
И к чему это?
>> No.28390 Reply
>>28371
Картинка наверное понравилась
>> No.28951 Reply
File: 1288018317940.jpg
Jpg, 394.23 KB, 850×1143 - Click the image to expand
edit Find source with google Find source with iqdb
1288018317940.jpg
Анон, а принцип суперпозиции это же наше предположение, ведь так? Тогда хотелось бы узнать, есть ли экспериментальное подтверждение этого предположения?
>> No.28952 Reply
>>28951

Раз пишут во всех учебниках очевидно это подтверждено опытом
>> No.29001 Reply
>>28952
Хорошо, тогда хотелось бы узнать какие именно эксперименты его подтверждают? Эксперимент по просмотру всех учебников не предлагать
>> No.29003 Reply
>>28951
В общем виде это не предположение, а закон. Или тебя касательно квантовой механики интересует?
>> No.29004 Reply
>> No.29014 Reply
File: 1292440130359.jpg
Jpg, 111.69 KB, 570×800 - Click the image to expand
edit Find source with google Find source with iqdb
1292440130359.jpg
>>29003
Что ты имеешь в виду под понятием "физический закон"? Чур в википедию не подглядывать Да, интересует именно квантовая механика. На картинке в >>28951, кстати, название книжечки как бы намекает

>>29004
Правильно ли я понял, что электрон в данном эксперименте может находиться в суперпозиции двух базисных состояний - (пролетел через левую щель, пролетел через правую щель)? Если так - то тогда я не понимаю, как данный эксперимент подтверждает, например, то утверждение, что пролетевший через преграду электрон находился в состоянии (0.3, 0.7)

И еще напрашивается вопрос - почему когда они поставили глаз к преграде получилось то же, что и для шариков? Что такое коллапс волновой функции? Физикам известен процесс поражающий его?
>> No.29015 Reply
>>29014
Закон - скурпулёзно проанализированное и упорядоченное, перепроверенное и выигравированное в анналах наук следствие из наблюдаемых явлений. Ясен хрен, закон обязательно имеет под собой экспериментальные доказательства. Вот и без википедии, няша.
А книжку я не читал, и так пикча маленькая, а у меня зрения -3, лол!
Ну, если квант. мех. - то это очень интересно. А если в общем - то да, это закон.
Откомменчу попозже.
>> No.29017 Reply
File: Л_2.pdf
Pdf, 0.36 KB, 595×842 - Click the image to get file
Л_2.pdf
>>29014
> Правильно ли я понял
Да.
> как данный эксперимент подтверждает, например, то утверждение, что пролетевший через преграду электрон находился в состоянии (0.3, 0.7)
Никак. Он не может подтверждать то, на что не нацелен. Здесь ситуация такая, что вероятности 1:1, подвинь щели - получи другое распределение.
> почему когда они поставили глаз к преграде получилось то же, что и для шариков
Потому что глаз только притворяется глазом. Ты вообще представляешь как работает отслеживающая аппаратура для объектов такого размера? А лучше почитай-ка пдфрилейтед, там всё очень доступно изложено, а мне лень писать тонны пасты.
>> No.29018 Reply
File: 1294136462947.jpg
Jpg, 43.90 KB, 496×384 - Click the image to expand
edit Find source with google Find source with iqdb
1294136462947.jpg
>>29014
А понял. Если у нас есть система с двумя базисными состояниями, то мы можем сказать, что она находится в состоянии (0.3, 0.7), если мы, например, провели 1000 экспериментов и примерно в 300 из них у нас выпало первое состояние, а в остальных 700 - второе. Но как это применить к опыту с двумя щелями и электроном пока не знаю.

Думал привести в пример монетку, но она как я понимаю может находиться только в трех состояниях - (0, 1), (1, 0), (0.5, 0.5).

Про коллапс пока читаю английскую википедию
>> No.29019 Reply
Простейший пример суперпозиции (пускай и экстраполированный на макромир) - кот Шрёда. Он и жив, и мёртв, т.к. до вскрытия коробки и выбора системой опр. состояния жив/мёртв (а это и есть коллапс волновой функции) - кот жив+мёртв одновременно.
Поправьте, если ошибаюсь, труе-физики.
>> No.29020 Reply
>>29017
> почитай-ка пдфрилейтед
Да уже не нужно, я понял как экспериментально подтвердить то, что система находится в суперпозиции состояний первый параграф >>29018 Про природу коллапса волновой функции там вроде не написано (хотя я только бегло просмотрел).
>> No.29021 Reply
>>29020
Написано. Только не цепляйся к слову "коллапс", это не термин, насколько я знаю.
>> No.29023 Reply
>>29020
Так-то оно может и так, но я хотел эксперимент. В случае кота - это, к примеру, 1000 коробок с котами Шредингера.

>>29021
Ты имеешь в виду то, что фотон, посредством которого мы и производим измерение, взаимодействует с электроном таким образом, что переводит его из состояния в суперпозиции (например (0.4, 0.6)) в одно из базисных состояний (например (0, 1))
>> No.29024 Reply
>>29023
Нет. Он просто размывает картину, а уж что он там во что переводит - вопрос сложный.
>> No.29026 Reply
>>29023
Есть вещи, которые просто являются выводами. Эта "суперпозиция" - скорее математическая модель, отображающая процесс, а не объект/конкретное явление. Так что тут сложнее с "пруфами" - либо воспринять модель как отображение эксперимента - хотя бы как логичную интерпретацию... Либо представить этот процесс в другом виде.
Сама же истинность суперпозиции не ставится под сомнение.
>> No.29027 Reply
http://ufn.ru/ru/articles/1989/3/l/
Посоны, не знаете, где можно достать эту книжечку? Судя по описанию - полезная вещь. В самом журнале я так полагаю только описание книги, да?
>> No.29031 Reply
>>29027
Библиотека в томсамомвузедгееёиздали.
можно подойти на кафедру тогосамоговуза, спросить ведёт-ли тотсамыйпреподователь у них до сих пор, а он 90 процентов ещо преподаёт,с просить у него про книгу
>> No.29036 Reply
Доброанон, всерьёз интересуюсь квантовой физикой, поэтому хочу поднять мат. аппарат - сейчас он на уровне интегралов обыкновенных.
Посоветуй годного матана - необходимый минимум для того, чтобы понимать математический язык квант. меха. Ну там, операторы, матрицы - через что лучше всего это на дому изучить? И насколько реально?
>> No.29039 Reply
>>29036
Игорь, ты?
>> No.29040 Reply
>>29039
Nope.
>> No.29056 Reply
Ну так чо аноны, есть годные мануалы по матану? Для самостоятельного изучения, ёпт.
>> No.29061 Reply
Д. Письменный "Конспект лекций по высшей математике"
>> No.29065 Reply
>>29061
Благодарю.
>> No.29084 Reply
Я правильно понимаю, что когда на тело действует нескомпенсированная сила, то оно приходит в движение?
>> No.29091 Reply
>>29084
Правильно.
>> No.29104 Reply
>>29091

Это хорошо.
>> No.29113 Reply
File: Timor-Sea-Oil-Spill-satellite-image-acquired-October-28-2009-closeup-Credit-NASA.jpg
Jpg, 113.40 KB, 720×732 - Click the image to expand
edit Find source with google Find source with iqdb
Timor-Sea-Oil-Spill-satellite-image-acquired-October-28-2009-closeup-Credit-NASA.jpg
Почему недостижим абсолютный нуль температуры? Почему гелий не кристализуется при абсолютном нуле?
Пяно нефти авансом.
>> No.29114 Reply
>>29113
1 способ объяснения: Потому что тогда полностью остановится движение молекул вещества. Температура - это средняя кинетическая энергия молекул.
2 способ объяснения: А ты знаешь определение идеального газа? Предельную температуру, при которой давление идеального газа обращается в нуль при фиксированном объеме или объем идеального газа стремится к нулю при неизменном давлении, называют абсолютным нулем температуры.
К абсолютному нулю можно только приблизиться, но достичь его нельзя.
>> No.29115 Reply
>>29114
*определение абсолютного нуля температуры
быстрофикс
>> No.29119 Reply
>>29114
Я, вобщем, прежде чем тут спрашивать заглянул в вики.
> Абсолютный нуль температуры — это минимальный предел температуры, которую может иметь физическое тело.
> В рамках применимости термодинамики абсолютный ноль на практике недостижим. Его существование и положение на температурной шкале следует из экстраполяции наблюдаемых физических явлений, при этом такая экстраполяция показывает, что при абсолютном нуле энергия теплового движения молекул и атомов вещества должна быть равна нулю, то есть хаотическое движение частиц прекращается, и они образуют упорядоченную структуру, занимая чёткое положение в узлах кристаллической решётки (жидкий гелий составляет исключение). Однако, с точки зрения квантовой физики при абсолютном нуле температуры существуют нулевые колебания, которые обусловлены квантовыми свойствами частиц и физического вакуума, их окружающего.
>> No.29120 Reply
Так вот, меня смущают эти квантовые колебания: в чем их отличие от тепловых?
И ситуация с гелием мне не понятна. Алсо, что будет с водородом при абсолютном нуле?
И еще, что означает "на практике не достижима"? Принципиально не достижима или не достижима при данном уровне развития техники?
>> No.29121 Reply
>>29120
Водород затвердевает вблизи нуля на отличненько же. Разумеется, при приличном давлении. Про которое, кстати, ты забываешь.
>> No.29125 Reply
>>29121
Гелий тоже, при давлении выше 25 атмосфер. Алсо, похоже что дело в том, что гелий дохуя инертный и не образует молекул вроде H2. Значит его частицы (атомы) весят меньше и более подвижны как одно связано с другим? и их сложнее упорядочить в кристаллическую решетку.
И все таки, если абсолютный нуль такой абсолютный, то почему у атомов гелия остается энергия, которая мешает кристаллизации?
>> No.29592 Reply
Доброанон, как распределить заряд по поверхности проводящей сферы, чтобы поле внутре было однородным?
>> No.29593 Reply
>>29592
А держать ты его руками будешь, чтобы он не убежал по проводнику?
>> No.29595 Reply
File: 1225483069_atom.jpg
Jpg, 17.04 KB, 500×500 - Click the image to expand
edit Find source with google Find source with iqdb
1225483069_atom.jpg
Аноны, объясните мне состав работы тока. Ну вот например ток проходя через электролит производит работу, по закону фарадея, при этом еще нагревает электролит, излучает радиоволны, притягивает находящийся рядом металлический предмет, выбивает электроны из собственного проводника и так далее и тупая пизда. В каких пропорциях распределяется энергия на производимые работы? У меня в учебнике даны формулы для каждого процесса отдельно, но про случай их одновременного осуществления ничего не сказано.
>> No.29596 Reply
>>29593
Силой мысли же!
Имел в виду поиск зависимости плотности заряда от полярного угла, конечно.
>> No.29597 Reply
>>29596
А тебе не кажется, что в равномерно заряженной сфере поле будет однородное?
>> No.29598 Reply
>>29597
Будет. Но так же неинтересно. А если б ещё ненулевое...
>> No.29603 Reply
>>29598
Так не бывает.
>> No.29606 Reply
>>29598
В электростатике с проводящей сферой ты такого не создашь, т.к. из граничных условий потребуется ненулевая напряженность поля внутри сферы.
>> No.29607 Reply
>>29595 Попадалась мне в школьные годы милая книжка "Зри в корень!" издательства "Квант". В ней были собраны разного рода задачки "с подвохом" на применение физики в быту. Так вот, в описании к последней, кажется, задаче говорилось, что в любой рассматриваемой системе действуют все законы физики, включая еще не открытые. И задача физика состоит в том, чтобы уметь построить приближение, которое будет давать результаты, достаточно близкие к экспериментальным (от себя добавлю, что в некоторых случаях точность хотя бы до порядка величины - уже успех). Оценивай вклад и сравнивай. В зависимости от условий эксперимента на первый план могут выходить те или иные эффекты.
>> No.29608 Reply
>>29607
> в описании
в комментарии
slowfix
>> No.29623 Reply
File: 1267674738997.jpg
Jpg, 43.20 KB, 250×208 - Click the image to expand
edit Find source with google Find source with iqdb
1267674738997.jpg
>>29607
Скорее всего местный анон не может понять моего сумбурного изложения. Ладно, тот же вопрос в более простом варианте: у газа, как известно, в идеальных условиях постоянно отношение PV/T. Если мы сжимаем газ поршнем в замкнутом пространстве, то у него вследствие уменьшения объема увеличится давление(это очевидно) и температура(на этом основан дизельный ДВС). В каких пропорциях работа затраченная на сжатие газа перейдет в давление и температуру? И от чего это зависит?
>> No.29628 Reply
>>29623
охуеть, ты замечательно связал електрику и термодинамику, я так не могу.
Сейчас уже обьяснить не смогу, но проходил это по курсу "термодинамика".
Помоему, зависимость отображается грфиками адиабат.
Погугли "Циклы ДВС", там должно быть 2 замкнутых цыкла из 2-х адиабат и 2-х изобар(?)/изохор, соответственно для каждого графика,тебе надо только одна ветка из графика, отвечающая за сжатие воздуха, да именно воздуха, дизель впрыскиваеться в воздушную смесь, когда та уже находится под невебичным давлением по ним можно узнать примерно отношения данных велечиноно не постоянно
>> No.29649 Reply
>>29623
>>29628
Если я не ошибаюсь, то
> В каких пропорциях работа затраченная на сжатие газа перейдет в давление и температуру
зависит от характера протекания процесса (изотермический и изобарный дадут соответственно полный «переход» работы в давление и в температуру соответственно).
>> No.29651 Reply
>>29649
> мы сжимаем газ поршнем
адиабата
>> No.29659 Reply
>>16169
потому что химия, а не физика
>> No.29660 Reply
>>29651
А где волшебное «в адиабатической оболочке?»
>> No.29662 Reply
>>29659
Химия — частный случай физики же. Более простая и удобная модель представления.
>> No.29668 Reply
>>29662
блеать, гуглим

1)физические свойства вещ-ва
2)химические свойства вещ-ва
>> No.29713 Reply
>>29668
ололо
>> No.29715 Reply
>>29659
А вот и нет, испарение влаги - физика.
>> No.29717 Reply
ХИМИЯБЛЯДИ СОСНУЛИ
>> No.29719 Reply
>>29717
Семён, что за хуйню ты несёш?
>> No.29720 Reply
>>29719
Я не семён, но вы и вправду соснулей. Алсо химия - быдлонаука.
>> No.29722 Reply
>>29720
>>29717
Не читал.

>>29659
поешь говна няша, и перестань прогуливать!
>> No.29724 Reply
>>29722
да, да , правь мои треды, вахтёришко
>> No.29742 Reply
АНУ СЪЕБАЛИ БЫСТРО, Я СКАЗАЛ БЫСТРО СЪБАЛИ ВСЕ НАХУЙ
засрали годный тред, мудаки
>> No.29751 Reply
>>29742
хуевый тред был! опу лень учебник один раз открыть
>> No.29756 Reply
>>29751
Причём тут блять оп? Оп его создал пол года назад и забыл, тут люди задавали интересующие их вопросы не притянутые за уши к горам формул, а просто для понимания физического смысла тех или иных процесов.
ты ньюфаг?
>> No.30239 Reply
Курс_теоретической_физики_(Ландафшиц)Т._5.ч.1_Статистическая_физика.jpg
Анон, я знаю, ты занимаешься теоретической физикой и прочитал много книг на эту тему. Мне интересно, где можно найти примеры решения задач? Не задачники, а именно примеры решения типовых задач, чтобы посмотреть и понять как решать более сложные задачи. Конкретно интересуют квантовая механика и статистическая физика.
>> No.30240 Reply
>>30239
Неистово двачую этого теоретика.
>> No.30248 Reply
>>30239
Ну мы, например, решали задачи по статфизике при помощи учебника Квасникова. Первый том - термодинамика, 2-3 статфиз. Там, собственно, текст теор. материала перемежается примерами решённых задач. Можно его глянуть.
>> No.48158 Reply
Реквестирую годные пособия по физике, дорогой анонимус.
>>48157-нищеброд-кун
>> No.48354 Reply
Здравствуйте, госопда физики и им сочувствующие.
Мне тут приспичило посчитать КПД зубчатых передач в трансмиссии. В связи с этим 2 вопроса.
1) Формула.
2) входные данные.
>> No.48366 Reply
>>48354
все формулы для таких сложных явлений как правило эмпирические, созданы на основе опытных данных. более того, часто кпд одного типа передач принимают постоянным, например коническая 0,96, цилиндрическая 0,98, червячная 0,7 и т.д. (цифры из головы). короче открывай учебник по деталям машин. но если уж заморачиваться, то есть такая отрасль как триботехника и там это изучается более подробно.
>> No.48378 Reply
File: 803caf87222efba38862292a78469876.png
Png, 87.85 KB, 400×400 - Click the image to expand
edit Find source with google Find source with iqdb
803caf87222efba38862292a78469876.png
>>30239
> и статистическая физика
Если кому-то ещё интересно, есть книжка Кубо, статистическая механика. Там где-то половина объёма - разбор задач. Но она далеко не entry level.
>> No.48498 Reply
>>48378
Заинтересовал. Где она есть?
>> No.48508 Reply
File: fords-theater-review-7013.jpg
Jpg, 44.08 KB, 513×449 - Click the image to expand
edit Find source with google Find source with iqdb
fords-theater-review-7013.jpg
>>48498
Запомни этот адрес, друг мой: http://bib.tiera.ru/

В строку поиска вбей без кавычек "Kubo statisticheskaja mehanika" (да-да, именно транслитом), и да пребудет с тобой счастье.

мимоаспирант
>> No.48914 Reply
>>48158
Немного вверх.
>> No.49021 Reply
Анон, привет, есть вопрос по заданию, не хотелось бы сюда писать ее условие, задачка простенькая, кто сможет истолковать? Я начал делать, но запутался.
>> No.49338 Reply
File: 240069387.jpg
Jpg, 57.69 KB, 600×500 - Click the image to expand
edit Find source with google Find source with iqdb
240069387.jpg
Доброанон, помоги. Завтра экзамен. Почти нифига не знаю физику. Так вот, анон, посоветуй мне годную литературу. Гугл дает много всякой всячины. Не знаю что и учить.
первокурсота-кун
с:чересчур читают
>> No.49339 Reply
>>49338
Сивухин, Матвеев, Берклеевский курс.

Алсо,
> Завтра экзамен.
> Почти нифига не знаю физику.
подсказать, какой результат сдачи наиболее вероятен?
>> No.49340 Reply
>>49339
Знаю. Но хотя бы на 3 написать. Тем более не полный 0 же
>> No.49350 Reply
File: Безымянный.png
Png, 7.09 KB, 508×492 - Click the image to expand
edit Find source with google Find source with iqdb
Безымянный.png
Добрый вечер, Анон. У меня возник вопрос по задаче. Толкнули цилиндр, он начал катится вверх, затем остановился, ну и понятное дело начал катиться вниз. Так вот, как определить время, сколько он катился до остановки? Я пробовал через энергию решать, но не правильно. Надо через кинематику.
>> No.49351 Reply
>>49350
Упс, извиняюсь за ошибочки.
>> No.49353 Reply
>>49350
g*sin(alpha)/V
>> No.49355 Reply
>>49353
не,не
>> No.49357 Reply
>>49355
Не слушай того анона (у него размерность не сходится даже).

(3/2)v/(gsin(alpha))
>> No.49358 Reply
>>49357
А можно постепенный расчет, просто я хочу разобраться к экзамену с такиим задачками?
>> No.49362 Reply
>> No.49363 Reply
>>49362
Не-не, я делал через закон сохранения, мне сказали не через него.
>> No.49365 Reply
File: DSC_0690.jpg
Jpg, 1271.82 KB, 2812×3519 - Click the image to expand
edit Find source with google Find source with iqdb
DSC_0690.jpg
Все, наконец-то разогнал повешенных на меня студентов. Лови решение.

USA-PhD-кун
>> No.49366 Reply
>>49365
Забыл добавить: для тренировки можешь решить эту же задачу, расписав моменты относительно оси C цилиндра. Выйдет немного проще (т. Гюйгенса-Штейнера не понадобится), но все же.
>> No.49368 Reply
>>49365
Ты в аспирантуру забугорную заканчивал?
Можешь запилить кулстори, как туда попал?
Мимопроходил
>> No.49388 Reply
>>49365
Ты просто шикарен, благодарю.
>> No.49408 Reply
>>49388
Что шикарного? Такие задачки в 10 классе решают.
>> No.49414 Reply
>>49408
Аноны, у меня есть проблема с задачками для вуза, там буквально объяснения решения не хватает, если закину, поможете текстом? Всего три пояснения.
>> No.49421 Reply
Звук создаётся в результате колебания какого-либо тела. Что колеблется, когда хрустит снег?
>> No.49422 Reply
>>49421
Снег.
>> No.49423 Reply
>>49421
> Звук создаётся в результате колебания какого-либо тела.
Нет, звук в воздухе создается в результате любого физического процесса, вызывающего возмущения воздуха. Сам процесс не обязан быть колебанием, несмотря на то, что в результате возмущения воздуха появляется колебание. В данном случае ломаются ледяные иголочки, и во время релаксации форма кусочков снежинки быстро меняется, воздействуя на воздух. Это воздействие вызывает колебания воздушной среды, соответствующее звуку.
>> No.49439 Reply
>>49414
Ладно, аноны, я завтра закину, помогите, молю. В качестве оплаты могу по реквесту нарисовать что-нибудь.
>> No.49440 Reply
>>49414
Ладно, аноны, я завтра закину, помогите, молю. В качестве оплаты могу по реквесту нарисовать что-нибудь.
>> No.49441 Reply
File: So.jpg
Jpg, 9.77 KB, 450×525 - Click the image to expand
edit Find source with google Find source with iqdb
So.jpg
Анон, я тут как всегда захотел замахнуться на святое и решил поделиться следующей задачкой: Как ты думаешь, могут ли существовать какие-нибудь гипотетические плюшки в ситуации с упругим столкновением двух тел, относительная скорость которых друг на встречу другу будет равна скорости распространения звука в веществе из которого сделаны эти тела?
пик намекнёт о каком роде плюшек я говорю
>> No.49514 Reply
>>49388
Рад помочь, бро хотя вот этот анон (>>49408), в общем-то, прав.

>>49368
Нет, я ее не закачивал. Прямо сейчас я в этом всем варюсь, иногда с кулфейсом, иногда матерясь про себя и попутно изображая улыбку на лице.

Стори запилить могу, но вот будет ли она кул? Не уверен. Если вкратце: технарь физик-теоретик, forever-aone, к концу бакалавриата понял, что жизнь идет куда-то не туда и надо срочно найтисебетян что-то менять, и тут обстоятельства подсказали вариант с зарубежной аспирантурой. После этого я пошел в магистратуру с целью оттянуть время и подтянуть английский. В начале второго курса магистратуры сдал международные экзамены, результаты (и копию бакалаврского) отослал в несколько вузов, на инженерные специальности. Мною заинтересовались в самом топовом и в самом низкоранговом университете из тех, с которыми связывался, в итоге от них я и получил admission. Поломавшись, согласился на предложение топового (один из старых и престижных университетов США). После этого я нехотя (меня ломало, я хотел уехать) защитил диплом, получил визу (не без приключений из-за т.н. Technology Alert List) - и вот я здесь.

Могу расписать интересующие вопросы подробнее, если хочешь, буду рад помочь чем могу, если тоже планируешь ехать - просто сейчас не хочу превращать тред в сборник копипасты из своей биографии.
>> No.49515 Reply
>>49514
> alone
fix ftgj
>> No.49516 Reply
>>49440
Куда пропал?
>> No.49522 Reply
File: ПБТ-13 кр2 - не зачтено.rar
Rar, 0.09 KB, 0 files - Click the image to get file
view
ПБТ-13 кр2 - не зачтено.rar
>>49516
В /r, просто тут все молчали, я и подумал, что забили.
>> No.49546 Reply
File: 1261072343021.gif
Gif, 288.55 KB, 750×750 - Click the image to expand
edit Find source with google Find source with iqdb
1261072343021.gif
>>49522
бумп
>> No.49548 Reply
File: arya-stark1.jpg
Jpg, 32.65 KB, 430×350 - Click the image to expand
edit Find source with google Find source with iqdb
arya-stark1.jpg
Господа физики, а можно ли сжимать воздух до твердого состояния и использовать его как оружие? Например пушка, у которой внутри создается большое давление, воздух сжимается и выстреливается в нужном направлении, пробивая там все вокруг.
>> No.49549 Reply
>>49548
Воздух -> Смесь газов-> Газы при сжатие ...Ну ты понел.
Но. С близкого расстояния может что-нибудь да и выйдет. Но с близкого.
>> No.49552 Reply
File: arya-stark2.jpg
Jpg, 27.15 KB, 430×350 - Click the image to expand
edit Find source with google Find source with iqdb
arya-stark2.jpg
>>49549
> Газы при сжатие ...Ну ты понел.
Увы, не понял. Я ж не физик, няша.
>> No.49553 Reply
>>49552
Я тоже не физик, но сжижается же.
мимодвоечник
>> No.49554 Reply
>>49548
Гугли Пушку Архимеда
>> No.49569 Reply
где-то читал, что в ядре юпитера предполагается наличие твердого водорода. ну а в земных условиях это конечно нереально.
>> No.49571 Reply
>>49569
Учи матчасть! Твердый гелий получили в лаборатории Кееза в 1926 году, а у гелия температура плавления ниже чем у водорода...
>> No.49583 Reply
>>49571
То есть пушка, стреляющая твердым гелием и истребляющая все вокруг, возможна?
>> No.49584 Reply
>>49583
В принципе да, но только на Юпитере :).
Твёрдый гелий при контакте с нормальными условиями начнёт резво расширяться и испаряться. До адресата долетит лишь лёгкий ветерок, если он не привязан к жерлу пушки. А если поместить твёрдый гелий в контейнер, то технически ты уже стреляешь контейнером, а не гелием.
>> No.49611 Reply
Аноны, так как, никто не поможет >>49522? Предупредите хоть.
>> No.49702 Reply
File: 16.png
Png, 75.80 KB, 1050×256
edit Find source with google Find source with iqdb
16.png
File: 13.png
Png, 83.73 KB, 1071×268
edit Find source with google Find source with iqdb
13.png

/u/, можешь оперативно в течение часа, если прошло больше времени, не напрягайся помочь с 2 задачами пикрелейтед. Никак не получается решить. последние 2 задания заочного тура олимпиады МИФИ
>> No.49704 Reply
>>49702
В первой k скорее всего равен тангенсу данного угла.
>> No.49791 Reply
В проводнике за время t = 10 с при равномерном возрастании силы тока от I1 = 1 А до I2 = 2 А выделилось количество теплоты Q = 5 кДж. Найти сопротивление R проводника.

Количество теплоты, выделяющееся в проводнике при прохождении тока за время dt равно, согласно закону Джоуля-Ленца dQ=I^2×R×dt.
Зависимость тока от времени линейная: I=kt+I1, где k=(I2-I1)/T.
Тогда полная теплота равна интегралу Q=∫▒dQ=∫_0^T▒〖I^2×R×dt〗
> следует объяснить необходимость применения интегрирования для нахождения теплоты.
Помоги, анон, что он имеет ввиду?
>> No.49793 Reply
>>49791

Он наркоман, очевидно же.

Напиши что-нибудь по поводу того, что в каждый момент выделяется разное количество теплоты, а потому мы должны просуммировать все элементарные приращения, что в пределе dt->0 дает интеграл. Но в любом случае такой коммент - это пиздец, полный пиздец. Преподаватель физики гоняет студентов на оформлении, а не непосредственном решении задач. Гнать таких в шею.
>> No.49795 Reply
File: Новый точечный рисунок.jpg
Jpg, 5.47 KB, 176×139 - Click the image to expand
edit Find source with google Find source with iqdb
Новый точечный рисунок.jpg
>>49793
Алсо, все вопросы у него того же типа.
Ещё, на ту же тему, просто я вообще не представляю, что менять:

Задача:
Электрон, пройдя ускоряющую разность потенциалов U=1,2 кВ, вошел в скрещенные под прямым углом однородные магнитное и электрическое поля и при этом скорость его движения не изменилась. Определите напряженность Е электрического поля, если магнитная индукция В поля равна 6 мТл.

Рисунок к ней пикрелейтед.

Коммент:
Электрон несёт на себе отрицательный заряд, а рисунок создан для движения в электромагнитном поле положительного заряда. Перестройте рисунок так, чтобы знаки проекций векторов на оси координат в решении задачи не изменились.
>> No.49797 Reply
>>49795
На рисунке поле E направлено вдоль OX, B - вдоль OY, V вдоль OZ. Соответственно, Fe направлена по оси OX, a Fл - против. Я не знаю, о проекциях каких векторов говорит твой препод-омич, но буду исxодить из предположения, что говорит он о силах - в этом случае тебе надо сменить ориентации полей E и B на противоположные

Bообще, чтобы не гадать почем зря, ты можешь напрямую спросить у него, что он имел в виду? В конце концов, это не твоя обязаннасть разгадывать такие-вот ребусы.
>> No.49798 Reply
>>49797
Дык это дистанционка и чаще всего препод громогласно молчит.
>> No.49799 Reply
>>49797
> тебе надо сменить ориентации полей E и B на противоположные
Т.е. местами поменять?
Прости за мою тупость, анон, просто совсем кэн нот ту физикс.
>> No.49800 Reply
>>49799
Нет, Е направить против оси OX, a B - против OY.
>> No.49802 Reply
File: 1111.jpg
Jpg, 4.89 KB, 238×218 - Click the image to expand
edit Find source with google Find source with iqdb
1111.jpg
>>49800
Т.е. вот так?
>> No.49936 Reply
>>49802
Реакция преподавателя.


Повторно. Ошибка! Электрон несёт на себе отрицательный заряд, а рисунок создан для движения в электромагнитном поле положительного заряда. Перестройте рисунок так, чтобы знаки проекций векторов на оси координат в решении задачи не изменились.

Суть расположения векторов не изменилась. Кулоновская сила должна действовать против силовых линий электрического поля на отрицательный заряд. Направление силы Лоренца определяется по правилу левой руки, но результат, выдаваемый этим правилом, для отрицательного заряда изменяется на противоположный (вектор F заменяется на –F).




Алсо, за прошлую подсказку спасибо, прошло. Но этот няша видимо хочет, чтобы я 100% работ сделал, хотя бы на тройку. Сейчас у меня верно зачтено 6 из 8.
>> No.49968 Reply
>>49936
> Направление силы Лоренца определяется по правилу левой руки кококококококо
Направление силы лоренца определяется как F_l = q[V x B] (где q может быть как положительным, так и отрицательным, 'x' - векторное произведение)
На твоем рисунке силы показаны правильно (единственное, у полей вектора не обозначены - у E направление влево, у B - вниз), и если уж твой препод полуебок, то ему уже к сожалению ничем не помочь. Левая рука у него, видите ли - у меня БИФУРКАЦИЯ БОГДАНОВА-ТАКЕНСА от таких преподавателей.
>> No.49974 Reply
>>49968

Анон, во-первых огромное тебе человеческое спасибо, что ты со мной возишься, без тебя бы мне шандец наступил. И это при условии, что остальные предметы я сдал на 4-5 и раньше срока. И только эта физика мешает мне закончить семестр.
Во-вторых - для того, чтобы рисунок стал идеальным, нужно вектора стрелочками (влево и вниз) над буковками обозначить?
Плюс - как покультурнее и научно будет звучать фраза "Вы наркоман, всё верно, так как и т.п"?
И третье - а как выглядит идеальный рисунок по его мнению?
>> No.49980 Reply
>>49974
Приду домой - нарисую и напишу.
>> No.49981 Reply
>>49980
Спасибо, бро.
>> No.50000 Reply
File: Untitled.jpg
Jpg, 906.93 KB, 3752×2979 - Click the image to expand
edit Find source with google Find source with iqdb
Untitled.jpg
>>49974
> для того, чтобы рисунок стал идеальным, нужно вектора стрелочками (влево и вниз) над буковками обозначить?
Значки вектора ("горизонтальная стрелочка над буквой") ставить надо обязательно, но я не про это говорил: буквы-то ты переставил, но сами вектора (направления полей) не нарисовал. И оси подпиши. Короче говоря, смотри пикрелейтед. Там же приведено доказательство того, что вектора нарисованы правильно, а не как в охуительных историях твоего преподавателя - нет, ну это надо же: один из пары векторов, формирующих векторное произведение, поменял направление, а "суть расположения векторов не изменилась". Чего несет?
> как покультурнее и научно будет звучать фраза "Вы наркоман, всё верно, так как и т.п"?
Как вариант:
Дляначалаприведицитатуизегослов,написавеезеленымичернилами - ноцитируйнедоконца:оборвинасерединепредложенияидобавь"кококококо", нутыпонел. Затемпишичто-нибудьнавроде "<преподнейм>, я считаю, что Вы ошибаетесь, утверждая, что при указанном направлении полей и скорости электрона направление векторов сил будет иным. В самом деле, запишем выражения для сил, действующих на движущийся заряд...", после чего приводишь доказательство с пикчи.
>> No.50001 Reply
>>50000
Спасибо, няша, сегодня-завтра напишу ему, потом отпишусь о результатах, когда будут.
Истинно доброчановский гет.
>> No.50020 Reply
>>50000
Что это за таблица 3 на 3?
>> No.50031 Reply
>>50020
Определитель, с помощью которого можно вычислять векторное произведение.
>> No.50033 Reply
>>50031
Это не может быть определителем.
>> No.50034 Reply
>>50033
lolsto?
>> No.50038 Reply
>>50034
Проще говоря, все элементы в записи определителя должны принадлежать некоторому полю, т.е. быть числами.
>> No.50039 Reply
>>50038
Если такая формальная запись работает и помогает быстро и правильно находить компоненты векторного произведения, то от нее не стоит отказываться только из-за того, что по определению это определителем быть не может. Хочешь соблюдения всех условностей - пожалуйста, перемножай с использованием тензора Леви-Чивиты, никто не мешает.
>> No.50069 Reply
Есть ли какой-нибудь хороший задачник с простыми школьными задачами для начала?
>> No.50087 Reply
>>50069
Н.И Гольдфарб. Там даже решения есть, насколько я помню.
http://www.alleng.ru/d/phys/phys36.htm
>> No.50403 Reply
>>50000
Зачел, когда я подробно срисовал твой рисунок, плюс доказательство, сославшись на то, что раньше рисунок не тру был.
Вот собсно, цитата:


Пояснение. Рисунок в предыдущей версии решения данной задачи отличается от настоящего. В этой версии рисунок правильный и направления сил действительно соответствуют движению отрицательного заряда. Рядом с ним в красной рамке вставлен рисунок из контрольной работы, которую я рецензировал раньше. Они отличаются. Вектор должен показываться стрелкой, а не просто буквой. При рассмотрении решения в предыдущий раз я считал, что напряжённость электрического поля направлена слева направо, а индукция магнитного поля – снизу вверх. В действительности всё оказалось наоборот, что и отражено на сегодняшнем рисунке. Создавайте рисунки так, чтобы они читались однозначно.



Спасибо, доброанон, лучей добра и любви тебе!
>> No.50404 Reply
>>16169
Расскажите про теорию струн господа професора.
>> No.50428 Reply
>>50403
Еще вопросы по этой или другим работам остались?
>> No.50431 Reply
>>50428
Ну, там он ещё к одной задаче прикопался, но принял и с ней, так что пока нет. У меня ему ещё две лабы на подходе, но там вроде просто не к чему придраться, ибо практические же.
>> No.50534 Reply
Искал тред рандомных вопросов, но не нашел. Спрошу тут.
Аноны, подскажите, какая температура в открытом космосе?
>> No.50539 Reply
>>50534
Температура чего? У пустоты нет температуры. А так - практически любая.
>> No.51565 Reply
File: 1325359726114.jpg
Jpg, 53.97 KB, 400×393 - Click the image to expand
edit Find source with google Find source with iqdb
1325359726114.jpg
Занесите кафедру в прикреплённый пост, а то ньюфаги не находят.
>> No.51566 Reply
>>51565
Ну пиздец. Я что ли виноват, что на этой борде нет полнотекстового поиска?
>> No.51572 Reply
>>51566
Мат под спойлер, няша
Гугл вроде как ищет то, что не ищется встроенным.
>> No.51573 Reply
>>51566
Ну я никого и не обвиняю, заметь. Просто напоминаю. Да к тому же не тебе, а человеку, который прикреплял ссылкотред.
>> No.51606 Reply
/r/ физик-куна. Суть вопроса: Есть сосуд с сжиженым воздухом. В сосуд вводится немного жидкой H2O. Что произойдёт? Они разделятся как вода и масло, или вода растворится в сжиженом газе как в спирте? Или она будет растворяться, но только до достижения определёной точки, а потом будет отделяться разделом фаз?
>> No.51612 Reply
File: 1329269685333.jpg
Jpg, 255.39 KB, 1000×727 - Click the image to expand
edit Find source with google Find source with iqdb
1329269685333.jpg
Анон, помоги объяснить увиденное.

Вопрос из Пёрышкина стоял следующим образом: если смочить два листочка бумаги, один -- водой, другой -- растительным маслом, то слипнутся ли они, если положить их друг на друга.

Я ответил нет, предположив, что молекулы воды притягиваются к друг другу сильнее, чем к молекулам масла, потом провёл испытание и они, конечно же, слиплись, иначе я бы здесь не спрашивал. :3
>> No.51631 Reply
>>51606
Жидкая Н2О быстро станет твёрдой, потому что воздух сжижается при очень низких температурах.
>> No.51676 Reply
>>51612
А с чего ты взял, что они друг другу как-то сильнее или слабее? В воде водородные связи, на границе их может и не быть, и кислород с успехом притягивается к водороду в жирах в масле. Ну или к чему там. Но, подозреваю, что ответ перышкин хотел другой.
>> No.51686 Reply
>>51676
Ёпт, задача из учебника для 7-го класса. Кого ни спрашивал, всех вводит в ступор. Так какой же ответ?
>> No.51724 Reply
Палю годноту.
>> No.51725 Reply
Забыл прикрепить ссылку.

http://www.youtube.com/user/EmpiricSchool
>> No.51727 Reply
Посоны-хардкорщики, расскажите, что в мире теор. физики нового? Надежды на Теорию Всего базируются на ололо-коллайдере или есть какие-то значимые движения в области годных теорий и гипотез?
угорел по физике-кун
>> No.51761 Reply
>>51612
На поверхности бумаги слой жидкостей тонкий, вода с маслом вполне могут образовать эмульсию. На листе с водой масло остается?
>> No.52085 Reply
File: images (1).jpg
Jpg, 8.57 KB, 298×169 - Click the image to expand
edit Find source with google Find source with iqdb
images (1).jpg
>>16169
Физикач, приветствую тебя! Учусь на программера. Понял что не моё, хоть и хлебно. Хочу в физики-ядерщики, поближе к исследованиям в области управляемого термоядерного синтеза. Реквестирую инфу по вузам, прохладные от работающих/учащихся/абитуриентов. Да да, меня забанили в Google.
>> No.52093 Reply
File: 12635071908651.jpg
Jpg, 693.51 KB, 5670×3685 - Click the image to expand
edit Find source with google Find source with iqdb
12635071908651.jpg
/r/ физик-куна. Суть вопроса: Есть сосуд с сжиженым воздухом. В сосуд вводится немного жидкой H2O. Что произойдёт? Они разделятся как вода и масло, или вода растворится в сжиженом газе как в спирте? Или она будет растворяться, но только до достижения определёной точки, а потом будет отделяться разделом фаз?
пикрандом
>> No.52099 Reply
>> No.52116 Reply
Когда на тело не действуют никакие силы7 Ведь на Земле всегда сила тяжести?
>> No.52117 Reply
>>52116
Ты имеешь в виду Ф=0? Тогда силу тяжести можно уравновесить противоположно направленой силой, чтобы их сложение давало 0. Однако не забывай, что Земля еще и вращается, так-что лучше это делать на полюсах.
>> No.52118 Reply
>>52117

Когда скомпенсированы это понятно, просто в одной из формулировок первого закона Ньютона есть такая фраза "когда на тело не действуют никакие силы".
>> No.52272 Reply
>>51631
А если воздух нагрет?
(да, высокое давление!)
>> No.52295 Reply
>>52085
МИФИ же! Главный ядерный институт страны.
>> No.52318 Reply
>>52085
> Реквестирую инфу по вузам, прохладные от работающих/учащихся/абитуриентов.
Удобряю. Тоже хотелось бы узнать побольше про МИФИ.
>> No.52328 Reply
>>52118

К.О. подсказывает, что если тело находится далеко от Земли, то сила земного притяжения на него не действует.
>> No.52434 Reply
Вот! Я нашёл ближайший подходящий тред. Ближе бы мне подошёл "тред идиотских вопросов", но он, видимо, ушёл в бамплимит.
Википедия знает ответ на вопрос "почему небо синее", но она не даёт ответа, почему в таком случае небо не фиолетовое. Так почему?
>> No.52438 Reply
>>52434
> Википедия знает ответ на вопрос "почему небо синее", но она не даёт ответа, почему в таком случае небо не фиолетовое. Так почему?
Потому что оно СИНЕЕ
>> No.52441 Reply
>>52438
Плохой ответ. Почему плохой? Потому что он ПЛОХОЙ.
Это из той же оперы.
>> No.52442 Reply
>>52441
>>52434
Совсем дурачок?
>> No.52443 Reply
>>52441
> Поскольку синий цвет находится на коротковолновом конце видимого спектра, он больше рассеивается в атмосфере, чем красный. Благодаря этому, если посмотреть на участок неба вне Солнца, мы увидим голубой свет — результат рассеяния солнечного излучения.
Что тебе здесь непонятно, тупка?
>> No.52444 Reply
>>52443
то что фиолетовый цвет тоже находится в коротковолновом конце видимого спектра. И даже ближе к концу, чем синий.
>> No.52454 Reply
>>52444
Небо рассеивает лучи всех видимых цветов, но степень рассеивания обратно пропорциональна длине волны в 4-й степени, то есть в видимом диапазоне максимальна для фиолетовых, синих и голубых лучей. Для глаза такая смесь кажется голубой.
>> No.52460 Reply
File: nipah_sun.jpg
Jpg, 42.59 KB, 873×451 - Click the image to expand
edit Find source with google Find source with iqdb
nipah_sun.jpg
>>52454
Какой хороший анон! Много знает. И немного прав. Но если подумать о спектральной энергетической светимости Солнца (лучше измерить), то окажется, что она на фиолетовом свете в два раза меньше.
YouTube: Higurashi ~ Takano Stole the Precious Thing
>> No.52463 Reply
>>52444
Спасибо, всё правильно за меня ответил.
>>52460
>>52454
То есть фиолетового просто мало. Тогда всё просто сходится, спасибо за разъяснения.
>> No.52480 Reply
File: 125622216207112.jpg
Jpg, 134.60 KB, 807×861
Your censorship settings forbid this file.
unrated
Экзамен в три дня. На все вопросы нашел ответы, кроме вот этих:

8. Опишите механизм появления поверхностных связанных зарядов при поляризации неполярных диэлектриков.


9. Две параллельные плоскости находятся на расстоянии 0,50 см друг от друга, На плоскостях равномерно распределены заряды с поверхностными плотностями 0,20 мкКл/м2 и -0,30 мкКл/м2. Определите разность потенциалов между плоскостями.

13. Опишите структуру магнитного поля кругового витка с током. Приведите формулы для вычисления магнитной индукции поля, созданного круговым витком с током.


15. Рамка площадью 400 см2 помещена в однородное магнитное поле индукцией 0,15 Тл так, что нормаль к рамке перпендикулярна линиям индукции. При какой силе тока на рамку будет действовать вращающий момент 20 мН۰м?

16. В однородном магнитном поле с магнитной индукцией 0,014 Тл находится прямой проводник длиной 8,0 см, расположенный перпендикулярно к линиям индукции. По проводнику течет ток 2,0 А, величина которого поддерживается неизменной. Под действием сил поля проводник переместился на расстояние 5,0 см. Найдите совершенную при этом работу.


Молю, доброанон, необходимо хотя бы ещё 2-3 решенных вопроса (их всего 18, но этот препод в прошлый раз отказывался мне тройку ставить даже при 75% решенных задач).
Как оказалось, совмещать дистанционку и работу - то ещё удовольствие.
>> No.52484 Reply
File: 1111.JPG
Jpg, 24.49 KB, 406×529 - Click the image to expand
edit Find source with google Find source with iqdb
1111.JPG
>>52480
Апдейт. Уже нужна только теория:

8. Опишите механизм появления поверхностных связанных зарядов при поляризации неполярных диэлектриков.
13. Опишите структуру магнитного поля кругового витка с током. Приведите формулы для вычисления магнитной индукции поля, созданного круговым витком с током.
>> No.52493 Reply
>>52484
> 8. Опишите механизм появления поверхностных связанных зарядов при поляризации неполярных диэлектриков.
По определению же. При поляризации в диэлектрике возникают диполи. У диэлектрика есть поверхности, на которых лежат концы диполей. Соответствующий заряд на поверхности - сумма зарядов концов диполей.
> Опишите структуру магнитного поля кругового витка с током.
ruwiki://Соленоид
>> No.52496 Reply
>>52480
Не знаю, чем меня привлекает девушка с твоего пика. Наверное своей доступностью.
>> No.52498 Reply
File: 48eb3dde0dccf6e33...
Jpg, 198.87 KB, 856×1050
edit Find source with google Find source with iqdb
48eb3dde0dccf6e33008fa3c803417a0152bcbea.jpg
File: 0288b5351756b5137...
Jpg, 289.21 KB, 1135×900
edit Find source with google Find source with iqdb
0288b5351756b5137f8512cab5b0ac82.jpg

>>52496
Лично мне она нравится своим усталым видом на некоторых пиках и мужским поведением, сразу представляется такая девушка-Байкен.
>>52493
> это односложная катушка цилиндрической формы, витки которой намотаны вплотную, а длина значительно больше диаметра. Характеризуется значительным соотношением длины намотки к диаметру оправки, что позволяет создать внутри катушки относительно равномерное магнитное поле.
Прокатит, добрый человек?
>> No.53431 Reply
Задача:
Балон объемом V1 содержит сжатый воздух при давлении P1 и температуре T1. Какой объем воды можно вытеснить из цистерны подводной лодки воздухом этого баллона если лодка находится на глубине H2, где температура T2 и атмосферное давление P2

Туплю и не понимаю как решать, помогите пожалуйста
>> No.54595 Reply
Такая задачка, господа. Есть платформа на 4-х колесах, масса всего этого чуда + груз = 100 кг. Вопрос: какую силу надо приложить, чтобы тащить это все со скоростью 5 м\с?
>> No.54638 Reply
>>54595
Условие неполное. Что с трением?
>> No.54781 Reply
Анон подскажи формулу, по которой можно расчитать радиус орбиты у положительно заряженого иона, который влетает в магнитное поле.
>> No.54807 Reply
>>54781
Сила Лоренца?
>> No.54819 Reply
>>54807
Разобрался уже, но всеравно спасибо.
>> No.54821 Reply
>>54819
Ну и молодец.
>> No.54952 Reply
File: 296_large.jpg
Jpg, 56.85 KB, 704×370 - Click the image to expand
edit Find source with google Find source with iqdb
296_large.jpg
Здравствуй, физикач.
У меня к тебе не совсем обычная просьба.
Я пишу говно-рассказик. Там есть некое непонятное явление, которое, однако, я не хочу объяснять ШМАГИЕЙ, поэтому прошу вас проконсультировать меня по паре вопросов.
>> No.54953 Reply
[Esper-raws] To Aru Kagaku no Railgun - 13 [BDrip 720p x264 FLAC].mkv_snapshot_03.35_[2012.02.29_18.52.11].jpg
>>54952
А где вопросы?
>> No.54956 Reply
>>54638
ну так трение качения можно выбрать самостоятельно, разве нет? по справочнику резина по сухому асфальту 0,5 - 0,8
>> No.54960 Reply
Здравствуй, дорогой Доброанон. Пишу в надежде на твою помощь.
Вобщем есть такая задача.
В 1921 г. немецкий физик К. Рамзауэр обнаружил аномальную «прозрачность» атомов
криптона для электронов с энергией E = 0,6 эВ. Этот эффект обусловлен волновыми свойствами электронов. Моделируя поле атома с помощью одномерной прямоугольной потенциальной
ямы глубиной U = 2,5 эВ, оцените радиус атома криптона.

Подскажи что-нибудь насчёт неё. Не знаю как подступиться.
>> No.54968 Reply
>>54956
>>54595
Платформа сферична и катится, вероятно, без трения.
Порешать попробую.
>> No.54969 Reply
>>54960
Сначала пиздеж не по делу. Во-первых, это туннелирование. То есть бегут электроны (волна) в направление оси x, встречают барьер (в нашем случае, видимо, яму), часть отражается от границ, часть проходит. То есть задача решается так. Строим потенциальную яму глубины 2,5 эв, а ширины 2R, пишем граничные условия на волновую функцию (очень похожие на оптические). Дальше, зная, что максимум плотноости (яма прозрачна) на 0.6 эв подбираем R (максимален модуль квадрата волновой функци).

Тут один неочевидный момент. Как это так мы не решаем уравнение Шредингера, а сразу пишем граничные условия. Ну, ты можешь сначала решить задачу для потенциала типа ступенька, потом для барьера, потом для ямы, и это полезно. Но в итоге ты все равно придешь к тем же граничным условиям. (Как я понимаю, самый очевидный их вывод: слева и справа разные гамильтонианы, а волновая функция и ее производная непрерывны.)

А по делу смотри лабника МФТИ за 3 курс по общей физике. Точно лаба такая была и там был вывод на пальцах совсем.
>> No.54971 Reply
>>54969
Спасибо тебе большое. Буду разбираться.
>> No.54973 Reply
>>54970
Перекатываемся.
>> No.54998 Reply
>>54968
но как это сферична и почему без трения? представь, что надо найти ту силу, как если бы мне пришлось толкать автомобиль весом 100 кг, со скоростью 5 м/с.


Password:

[ /tv/ /rf/ /vg/ /a/ /b/ /u/ /bo/ /fur/ /to/ /dt/ /cp/ /oe/ /bg/ /ve/ /r/ /mad/ /d/ /mu/ /cr/ /di/ /sw/ /hr/ /wh/ /lor/ /s/ /hau/ /slow/ /gf/ /vn/ /w/ /ma/ /azu/ /wn/ ] [ Main | Settings | Bookmarks | Music Player ]